File:  [Local Repository] / db / baza / rusvuz17.txt
Revision 1.1: download - view: text, annotated - select for diffs - revision graph
Fri Nov 10 00:13:03 2017 UTC (6 years, 7 months ago) by rubashkin
Branches: MAIN
CVS tags: HEAD
*** empty log message ***

    1: Чемпионат:
    2: Открытый чемпионат вузов России по ЧГК. 2016/17
    3: 
    4: Дата:
    5: 00-000-2016
    6: 
    7: Тур:
    8: Первый игровой день. 1 тур
    9: 
   10: Дата:
   11: 30-Sep-2016
   12: 
   13: Редактор:
   14: Илья Иванов (Пермь)
   15: 
   16: Инфо:
   17: Редактор благодарит за помощь в подготовке пакета Виктора Абдураманова
   18: (Березники), Арсэна Атнагулова (Уфа), Олега Белослудцева, Константина
   19: Костенко, Вадима Опутина, Александра Пономарёва, Александра Шишкина,
   20: Юлию Шучалову и Дениса Яруллина (все - Пермь), Александра Быкова и Ирину
   21: Пинегину (оба - Киров), Андрея Ефремова (Могилев), Андрея Кокуленко
   22: (Омск), Александра Мудрого (Черновцы), Константина Науменко (Киев),
   23: Ксению Рудич (Москва), Константина Сахарова (Ивантеевка), Ульяну
   24: Фабричнину (Санкт-Петербург), а также Дмитрия Башука и команду "От Винта
   25: - Братья по фазе" (Харьков).
   26: 
   27: Вопрос 1:
   28: В пражском ресторане "Vytopna" [вЫтопна] процесс доставки пива
   29: механизирован: клиенты получают свое пиво, когда происходит ЭТО. Кто
   30: создал "ЭТО" в конце XIX века?
   31: 
   32: Ответ:
   33: [Братья] Люмьер.
   34: 
   35: Комментарий:
   36: За доставку пива в ресторане отвечает миниатюрная железная дорога -
   37: поезд с бокалом подъезжает к посетителю, и тот может забрать стакан.
   38: "Прибытие поезда" - знаменитый фильм братьев Люмьер.
   39: 
   40: Источник:
   41:    1. http://luxlux.net/prazhskiy-restoran-vytopna-poezd-vmesto-ofitsianta-35437/
   42:    2. http://ru.wikipedia.org/wiki/Прибытие_поезда_на_вокзал_Ла-Сьота
   43: 
   44: Автор:
   45: Игорь Демьянцев (Гомель)
   46: 
   47: Вопрос 2:
   48: В американском комедийном мультсериале сотрудники одной организации
   49: используют фургон, предлагающий свежие буррито. Назовите эту
   50: организацию.
   51: 
   52: Ответ:
   53: ФБР.
   54: 
   55: Зачет:
   56: FBI; Федеральное Бюро Расследований; Federal Bureau of Investigation.
   57: 
   58: Комментарий:
   59: Спецслужбы часто используют фургоны для слежки. В мультсериале агенты
   60: ФБР (по-английски - FBI) умудрились использовать фургон с надписью
   61: "Fresh Burritos Instantly" [фреш буррИтос Инстантли] ("Свежие буррито
   62: немедленно"), сокращенно - FBI.
   63: 
   64: Источник:
   65: Мультсериал "Симпсоны", s25e01.
   66: 
   67: Автор:
   68: Илья Иванов (Пермь)
   69: 
   70: Вопрос 3:
   71: В 1894 году астроном Эндрю Дуглас предложил построить обсерваторию в
   72: городке Флагстафф в Аризоне. Рассказывают, что в конце жизни Дуглас
   73: признался, что во Флагстаффе его интересовали только луны. В одном из
   74: слов предыдущего предложения мы пропустили две буквы. Напишите это слово
   75: в исходном варианте.
   76: 
   77: Ответ:
   78: Салуны.
   79: 
   80: Комментарий:
   81: Территория славилась своими салунами - Аризона была частью дикого
   82: Запада, а дело было в конце XIX века. Вот такой вот "научный" критерий
   83: выбора для обсерватории.
   84: 
   85: Источник:
   86:    1. http://ru.wikipedia.org/wiki/Обсерватория_Лоуэлла
   87:    2. http://www.peoples.ru/science/astronomy/andrew_ellicott_douglas/
   88: 
   89: Автор:
   90: Илья Иванов (Пермь)
   91: 
   92: Вопрос 4:
   93: По несколько парадоксальному утверждению комментатора, эксцентричный
   94: Фабьен Бартез иногда устраивал в воротах ЕЕ. Назовите ЕЕ словом с
   95: удвоенной согласной.
   96: 
   97: Ответ:
   98: Буффонада.
   99: 
  100: Комментарий:
  101: Бартез в воротах постоянно "чудил". Джанлуиджи Буффон - другой
  102: легендарный футбольный голкипер.
  103: 
  104: Источник:
  105: Трансляция финала чемпионата мира по футболу 1998 года Франция -
  106: Бразилия на телеканале "Матч-ТВ", 08.07.2016 г.
  107: 
  108: Автор:
  109: Илья Иванов (Пермь)
  110: 
  111: Вопрос 5:
  112: [Ведущему: обязательно произнести название игры полностью!]
  113:    В начале игры "Шерлок Холмс: Преступления и наказания" главный герой,
  114: стреляя по вазам, приветствует инспектора Лестрейда, чем изрядно
  115: удивляет последнего. Какие три слова мы пропустили в этом вопросе?
  116: 
  117: Ответ:
  118: С завязанными глазами.
  119: 
  120: Зачет:
  121: С закрытыми глазами; другие синонимичные ответы из трех слов.
  122: 
  123: Комментарий:
  124: Несмотря на завязанные глаза, Холмс определяет, что в комнату вошел
  125: именно инспектор Лестрейд, - главным образом, благодаря бряцанью
  126: наручников, звуку шагов и тому, что "скрипучая" ступенька не скрипнула
  127: (значит, гость бывает в доме часто).
  128: 
  129: Источник:
  130: Игра "Sherlock Holmes: Crimes and Punishments".
  131: 
  132: Автор:
  133: Илья Иванов (Пермь)
  134: 
  135: Вопрос 6:
  136: Герой фильма "Космос как предчувствие" знакомится в поезде с молодым
  137: летчиком Юрием. Когда тот направляется к выходу, наблюдательный герой
  138: окликает его и указывает на один из НИХ. Назовите ИХ.
  139: 
  140: Ответ:
  141: Шнурки.
  142: 
  143: Комментарий:
  144: Создатели фильма явно решили сделать отсылку к известной истории, когда
  145: у Гагарина, шедшего по аэродрому Внуково к правительственной трибуне,
  146: развязался шнурок. Впрочем, существует версия, что это были подтяжки для
  147: носков - однако в фильме, конечно, фигурировали именно шнурки.
  148: 
  149: Источник:
  150:    1. Х/ф "Космос как предчувствие" (2005), реж. Алексей Учитель.
  151:    2. http://www.retroportal.ru/gagarin_14_04_1961.shtml
  152: 
  153: Автор:
  154: Илья Иванов (Пермь)
  155: 
  156: Вопрос 7:
  157: Герой французской комедии любезно отдает молодой немке солонку и
  158: перечницу. Недовольная жена язвительно спрашивает, почему же он не
  159: решился предложить девушке арию. В одном из слов предыдущего предложения
  160: мы пропустили шесть букв. Напишите это слово в исходном варианте.
  161: 
  162: Ответ:
  163: Лотарингию.
  164: 
  165: Комментарий:
  166: Лотарингия - знаменитая провинция, которая (вместе с Эльзасом) долгое
  167: время была причиной ожесточенных сражений и споров между Францией и
  168: Германией.
  169: 
  170: Источник:
  171: Х/ф "Каникулы маленького Николя" (2014), реж. Лоран Тирар.
  172: 
  173: Автор:
  174: Илья Иванов (Пермь)
  175: 
  176: Вопрос 8:
  177: В уже упомянутой нами игре на стене в комнате Шерлока Холмса можно
  178: увидеть ИКС, написанный в 1872 году. Назовите ИКС двумя словами.
  179: 
  180: Ответ:
  181: Портрет Достоевского.
  182: 
  183: Комментарий:
  184: Игра называется "Шерлок Холмс: Преступления и наказания", что является
  185: отсылкой к знаменитому роману Федора Михайловича.
  186: 
  187: Источник:
  188:    1. Игра "Sherlock Holmes: Crimes and Punishments".
  189:    2. https://www.wikiart.org/ru/vasiliy-perov/portret-pisatelya-fedora-mikhaylovicha-dostoevskogo-1872
  190: 
  191: Автор:
  192: Илья Иванов (Пермь)
  193: 
  194: Вопрос 9:
  195: На карикатуре Жана Эффеля Бог, прогуливающийся среди хаоса, замечает,
  196: что мир - это ИКС. Одним из лучших ИКСОВ 2015 года был признан
  197: "нипричёмыш". Какое слово греческого происхождения мы заменили ИКСОМ?
  198: 
  199: Ответ:
  200: Неологизм.
  201: 
  202: Комментарий:
  203: На карикатуре, посвященной сотворению мира, Бог достаточно справедливо
  204: называет создаваемый им мир неологизмом. "Нипричёмыш" - скучный, тихий
  205: человек, всегда держащийся подальше от любых волнений, - был признан
  206: Экспертным советом при Центре творческого развития русского языка одним
  207: из лучших неологизмов 2015 года.
  208: 
  209: Источник:
  210:    1. Ж. Эффель. Сотворение мира. - М.: Изобразительное искусство, 1989.
  211:    2. https://snob.ru/profile/27356/blog/101993
  212: 
  213: Автор:
  214: Виктор Абдураманов (Березники)
  215: 
  216: Вопрос 10:
  217: По анекдотической версии Александра Экстера, жена однажды увидела
  218: картину мужа из цикла работ о женской бане и пришла в ярость. Какое
  219: произведение, согласно этой версии, было создано в результате?
  220: 
  221: Ответ:
  222: "Черный квадрат".
  223: 
  224: Комментарий:
  225: И в ярости замазала полотно черной краской. Так Малевич и придумал свой
  226: "Черный квадрат" (во что, конечно, верится с трудом).
  227: 
  228: Источник:
  229: http://www.vokrugsveta.ru/article/205155/
  230: 
  231: Автор:
  232: Илья Иванов (Пермь)
  233: 
  234: Вопрос 11:
  235: Одна из шведских церквей недавно вывесила объявление с напоминанием, что
  236: по всё еще действующему закону 1280 года никто не может быть взят в плен
  237: в церкви. Отдельно подчеркнуто, что под защиту закона подпадают и ОНИ.
  238: Назовите ИХ одним словом.
  239: 
  240: Ответ:
  241: Покемоны.
  242: 
  243: Комментарий:
  244: Судя по всему, любители поохотиться на покемонов делали это и в церкви.
  245: 
  246: Источник:
  247: http://www.diary.ru/~Logovojuni/p209907950.htm
  248: 
  249: Автор:
  250: Константин Костенко (Гремячинск - Пермь)
  251: 
  252: Вопрос 12:
  253: Опера ДжАкомо Пуччини "ТОска" заканчивается прыжком главной героини с
  254: высокой башни. По легенде, конец одной из постановок "ТОски" привел
  255: публику в восторг. Причиной этого стало использование ЕГО. Назовите ЕГО
  256: одним словом.
  257: 
  258: Ответ:
  259: Батут.
  260: 
  261: Комментарий:
  262: Исполнительница главной роли спрыгнула из декорационной башни, однако
  263: вместо привычных матрасов кто-то додумался установить батут. В итоге
  264: певица три или четыре раза подпрыгнула на нем, что не осталось
  265: незамеченным публикой.
  266: 
  267: Источник:
  268: Э. Ланди. Тайная жизнь великих композиторов.
  269: http://www.flibusta.is/b/390922/read
  270: 
  271: Автор:
  272: Илья Иванов (Пермь)
  273: 
  274: Тур:
  275: Первый игровой день. 2 тур
  276: 
  277: Дата:
  278: 30-Sep-2016
  279: 
  280: Редактор:
  281: Андрей Ефремов (Могилев)
  282: 
  283: Инфо:
  284: Редактор благодарит за помощь в тестировании Валерия Семёнова, Сергея
  285: Дубелевича и команду "Солянка" (все - Минск), команду "От винта - Братья
  286: по фазе" (Харьков) и Лидию Иоффе (Хайфа).
  287: 
  288: Вопрос 1:
  289: В научно-популярном фильме параллельная эволюция вирусов и человеческого
  290: организма сравнивается с НЕЙ. Назовите ЕЕ двумя словами, начинающимися
  291: на соседние буквы алфавита.
  292: 
  293: Ответ:
  294: Гонка вооружений.
  295: 
  296: Комментарий:
  297: Вирусы и человеческие клетки на протяжении тысячелетий вынуждены были
  298: изобретать всё новые и новые механизмы атаки и защиты, обмана и его
  299: распознавания.
  300: 
  301: Источник:
  302: Эфир телеканала "Наука 2.0", сериал "Тайная вселенная", серия
  303: "Путешествие внутрь клетки", эфир от 11.02.2015 г.
  304: 
  305: Автор:
  306: Андрей Ефремов (Могилев)
  307: 
  308: Вопрос 2:
  309: Автор исторического романа сравнил ЕЕ с заточённой в монастырь царицей,
  310: которую ветреный и капризный властелин променял на холодную,
  311: змеиноглазую разлучницу. Назовите ЕЕ одним словом.
  312: 
  313: Ответ:
  314: Москва.
  315: 
  316: Комментарий:
  317: Как известно, в XVIII веке Петр I перенес столицу в Санкт-Петербург.
  318: 
  319: Источник:
  320: Б. Акунин. Любовница смерти. http://www.flibusta.is/b/500333/read
  321: 
  322: Автор:
  323: Андрей Ефремов (Могилев)
  324: 
  325: Вопрос 3:
  326: В статье об ИКСЕ упоминаются коронарные артерии. В произведении XIV века
  327: обитатели ИКСА стараются прикрыть один бок другим. Назовите ИКС двумя
  328: словами.
  329: 
  330: Ответ:
  331: Третий круг.
  332: 
  333: Комментарий:
  334: Третий круг кровообращения обслуживает само сердце. Во втором
  335: предложении речь идет о третьем круге ада в описании Данте, в который
  336: помещены чревоугодники, стыдящиеся своей полноты. Вопрос в туре идет
  337: третьим - это дополнительная подсказка.
  338: 
  339: Источник:
  340:    1. http://www.science-education.ru/34-1342
  341:    2. https://otvet.mail.ru/question/47811228
  342: 
  343: Автор:
  344: Андрей Ефремов (Могилев)
  345: 
  346: Вопрос 4:
  347: На карикатуре, действие которой происходит весной, ПЕРВЫЕ испуганно
  348: смотрят на появляющиеся из земли ВТОРЫЕ. Какие однокоренные слова мы
  349: обозначили как "ПЕРВЫЕ" и "ВТОРЫЕ"?
  350: 
  351: Ответ:
  352: Снеговики, подснежники.
  353: 
  354: Источник:
  355: http://pikabu.ru/story/predvestniki_smerti_1944655
  356: 
  357: Автор:
  358: Андрей Ефремов (Могилев)
  359: 
  360: Вопрос 5:
  361: В одном из стихотворений Владимир Маяковский удивляется: неужели "зуб
  362: революций ступился о НИХ"? Какая денежная единица получила свое название
  363: от НЕЕ?
  364: 
  365: Ответ:
  366: Крона.
  367: 
  368: Комментарий:
  369: ОНА - это корона. Как известно, прокатившаяся по Европе волна революций
  370: способствовала падению не одной монархии.
  371: 
  372: Источник:
  373:    1. В.В. Маяковский. Радоваться рано.
  374: http://www.feb-web.ru/feb/mayakovsky/texts/ms0/ms2/ms2-016-.htm
  375:    2. http://ru.wikipedia.org/wiki/Крона_(денежная_единица)
  376: 
  377: Автор:
  378: Андрей Ефремов (Могилев)
  379: 
  380: Вопрос 6:
  381: В вопросе словом "ИКС" заменено другое слово.
  382:    Однажды в компьютерной игре "Герои меча и магии - 5" автору вопроса
  383: встретился ИКС дьяволят. Назовите ИКС, созданный в 1831 году королевским
  384: приказом.
  385: 
  386: Ответ:
  387: [Французский] Иностранный [легион].
  388: 
  389: Комментарий:
  390: Словосочетание "легион дьяволят" отсылает к цитате из Евангелия "Имя мне
  391: - легион". Иностранный легион был создан по приказу Луи-Филиппа I, а
  392: Почетный легион - еще при Наполеоне I.
  393: 
  394: Источник:
  395:    1. ЛОАВ (пользовательская карта "Живее всех живых").
  396:    2. http://ru.wikipedia.org/wiki/Имя_мне_%E2%80%94_легион
  397:    3. http://ru.wikipedia.org/wiki/Французский_Иностранный_легион
  398: 
  399: Автор:
  400: Андрей Ефремов (Могилев)
  401: 
  402: Вопрос 7:
  403: В сентябре 1957 года сразу после аварии на химкомбинате "Маяк" в
  404: челябинских газетах появились сообщения о НЕМ. ОНО бывает двух видов,
  405: которые на самом деле вовсе не симметричны. Назовите ЕГО двумя словами,
  406: начинающимися на разные буквы.
  407: 
  408: Ответ:
  409: Полярное сияние.
  410: 
  411: Комментарий:
  412: Советские газеты пытались дезинформировать население, выдав техногенную
  413: катастрофу за природное явление. До недавнего времени считалось, что
  414: полярные сияния в северном и южном полушарии являются симметричными.
  415: Однако одновременное наблюдение полярного сияния в мае 2001 года из
  416: космоса со стороны северного и южного полюсов показало, что северное и
  417: южное сияния существенно отличаются друг от друга.
  418: 
  419: Источник:
  420:    1. http://ru.wikipedia.org/wiki/Кыштымская_авария
  421:    2. http://ru.wikipedia.org/wiki/Полярное_сияние
  422: 
  423: Автор:
  424: Андрей Ефремов (Могилев)
  425: 
  426: Вопрос 8:
  427: Описывая шумную толпу в лондонском порту, Филип Депуа пишет, что
  428: невидимая ОНА поднималась до самого неба. Назовите ЕЕ двумя словами.
  429: 
  430: Ответ:
  431: Вавилонская башня.
  432: 
  433: Комментарий:
  434: Писатель упоминает "разноязыкий гомон". Слово "толпа" в вопросе -
  435: подсказка.
  436: 
  437: Источник:
  438: Ф. Депуа. Тайна короля Якова. http://www.flibusta.is/b/254925/read
  439: 
  440: Автор:
  441: Андрей Ефремов (Могилев)
  442: 
  443: Вопрос 9:
  444: При создании ИХ в резиденции сёгуна гвозди располагали в виде
  445: перевернутой буквы V [ви] для имитации щебета различных птиц, по
  446: которому можно было достаточно точно определить местонахождение
  447: посетителей. Назовите ИХ двумя словами, начинающимися на одну и ту же
  448: букву.
  449: 
  450: Ответ:
  451: Поющие полы.
  452: 
  453: Зачет:
  454: Птичьи полы.
  455: 
  456: Комментарий:
  457: Поющие (соловьиные) полы были созданы в замке сёгунов в Киото, в первую
  458: очередь, для того чтобы враги не могли пробраться туда незамеченными. В
  459: зависимости от того, голос какой именно птицы был слышен, можно было
  460: точно определить местонахождение проникнувшего в замок.
  461: 
  462: Источник:
  463: https://historyporn.dirty.ru/poiushchie-poly-na-strazhe-siogunov-540719/
  464: 
  465: Автор:
  466: Андрей Ефремов (Могилев)
  467: 
  468: Вопрос 10:
  469: Престарелый герой рассказа Стивена Кинга сравнивает свое тело с
  470: разрушенным ИКСОМ. Самый высокий ИКС был создан в Майами. Назовите ИКС.
  471: 
  472: Ответ:
  473: Замок из песка.
  474: 
  475: Зачет:
  476: Синонимичные ответы.
  477: 
  478: Комментарий:
  479: Известно выражение "песок сыплется". Город Майами известен своими
  480: пляжами. Кстати, предыдущий вопрос тоже был о зАмке.
  481: 
  482: Источник:
  483:    1. С. Кинг. Человек в черном костюме.
  484: http://www.flibusta.is/b/182301/read
  485:    2. http://lenta.ru/news/2015/10/27/zamok/
  486: 
  487: Автор:
  488: Андрей Ефремов (Могилев)
  489: 
  490: Вопрос 11:
  491: Блиц.
  492:    На одном специализированном обучающем сайте есть цикл детских
  493: стихотворений.
  494:    1. В одном из стихотворений ОНА "словно подлодка, стреляет прямой
  495: наводкой". Назовите ЕЕ одним словом.
  496:    2. В одном из стихотворений ОН предпочитает рукопашную. Назовите ЕГО
  497: одним словом.
  498:    3. В одном из стихотворений ОНА решает, кем стать. Назовите ЕЕ одним
  499: словом.
  500: 
  501: Ответ:
  502:    1. Ладья.
  503:    2. Король.
  504:    3. Пешка.
  505: 
  506: Комментарий:
  507: В шахматах ладья контролирует вертикаль и горизонталь, король бьет
  508: только соседние клетки, а пешка, дойдя до последней горизонтали, может,
  509: в зависимости от решения игрока, превратиться в другую фигуру.
  510: 
  511: Источник:
  512:    1. http://fenix64.com/stix-o-lade/
  513:    2. http://fenix64.com/stix-pro-korolya-2/
  514:    3. http://fenix64.com/stix-pro-peshku/
  515: 
  516: Автор:
  517: Андрей Ефремов (Могилев)
  518: 
  519: Вопрос 12:
  520: Решительно настроенный президент США Уильям Тафт считал, что "ЭТО" может
  521: успокоить забастовку рабочих. Обычно за один сеанс ЭТОГО устанавливают
  522: от четырех до двенадцати... Кого?
  523: 
  524: Ответ:
  525: Пиявок.
  526: 
  527: Комментарий:
  528: Речь идет о кровопускании. Тафт ратовал за применение силы в отношении
  529: бастующих.
  530: 
  531: Источник:
  532:    1. http://ru.wikipedia.org/wiki/Тафт,_Уильям_Говард
  533:    2. http://ru.wikipedia.org/wiki/Кровопускание
  534: 
  535: Автор:
  536: Андрей Ефремов (Могилев)
  537: 
  538: Тур:
  539: Первый игровой день. 3 тур
  540: 
  541: Дата:
  542: 30-Sep-2016
  543: 
  544: Редактор:
  545: Дмитрий Башук (Харьков)
  546: 
  547: Инфо:
  548: Редактор благодарит за помощь в подготовке вопросов команду "От винта -
  549: Братья по фазе" (Харьков).
  550: 
  551: Вопрос 1:
  552: Шутники утверждают, что для грузчиков характерно раздельное питание.
  553: Когда они разгружают картошку, то едят картошку, когда разгружают сыр -
  554: едят сыр. А когда разгружают кирпич, то это - ОН. Назовите ЕГО двумя
  555: словами.
  556: 
  557: Ответ:
  558: Разгрузочный день.
  559: 
  560: Комментарий:
  561: "Разгрузочный день", как и "раздельное питание", - устойчивое понятие в
  562: диетологии.
  563: 
  564: Источник:
  565: "Арт-Мозаика", 2016, N 34.
  566: 
  567: Автор:
  568: Дмитрий Башук (Харьков)
  569: 
  570: Вопрос 2:
  571: Одну из актрис Фаина Раневская причислила к отряду молеобразных и
  572: объяснила это тем, что у той все мысли только об АЛЬФАХ. Стихотворение
  573: Семена Кирсанова об АЛЬФЕ заканчивается словами "... чтоб не мог мороз
  574: ущипнуть тебя". Назовите АЛЬФУ.
  575: 
  576: Ответ:
  577: Шуба.
  578: 
  579: Источник:
  580:    1. Ф.Г. Раневская. Мудрые остроты Раневской.
  581: http://www.flibusta.is/b/380824/read
  582:    2. С.И. Кирсанов. Нащот шубы.
  583: http://www.flibusta.is/b/343461/read#t21
  584: 
  585: Автор:
  586: Дмитрий Башук (Харьков)
  587: 
  588: Вопрос 3:
  589: Для десятилетнего Нила Сети, исполнившего роль Маугли в недавнем фильме
  590: "Книга джунглей", костюмеры изготовили двадцать вариантов ИХ, которые
  591: меняли в зависимости от эпизода. Назовите ИХ двумя словами.
  592: 
  593: Ответ:
  594: Набедренные повязки.
  595: 
  596: Комментарий:
  597: Других костюмов Маугли и не надо. :-)
  598: 
  599: Источник:
  600: "Телескоп", 2016, N 16.
  601: 
  602: Автор:
  603: Дмитрий Башук (Харьков)
  604: 
  605: Вопрос 4:
  606: У основания кафедры венского собора Святого Стефана есть скульптурное
  607: изображение человека с циркулем в руке. Назовите это изображение словом,
  608: происходящим от древнегреческого и французского слов.
  609: 
  610: Ответ:
  611: Автопортрет.
  612: 
  613: Комментарий:
  614: (pic: 20160953.jpg)
  615:    Создатель кафедры каменотёс Антон Пильграм изобразил себя взирающим
  616: на дело своих рук; слово "автопортрет" происходит от древнегреческого
  617: "autos" и французского "portrait".
  618: 
  619: Источник:
  620:    1. Е.Н. Грицак. Вена. http://www.flibusta.is/b/116707/read
  621:    2. http://ru.wiktionary.org/wiki/автопортрет
  622: 
  623: Автор:
  624: Дмитрий Башук (Харьков)
  625: 
  626: Вопрос 5:
  627: [Ведущему: при чтении явно указать запятую после первого пропуска.]
  628:    Опальный поэт из книги Артуро Переса-Реверте в одном из эпизодов так
  629: каламбурит о себе: "Дела идут [пропуск], дела [пропуск] дышат...".
  630: Заполните пропуски - двумя словами каждый.
  631: 
  632: Ответ:
  633: "... на лад...", "... на ладан...".
  634: 
  635: Комментарий:
  636: Такой вот грустный каламбур.
  637: 
  638: Источник:
  639: А. Перес-Реверте. Капитан Алатристе.
  640: http://www.flibusta.is/b/456025/read
  641: 
  642: Автор:
  643: Дмитрий Башук (Харьков)
  644: 
  645: Вопрос 6:
  646: Материал журнала "Вокруг света" под заголовком "Личное влияние" посвящен
  647: ИМ. Кстати, одна из ИХ разновидностей называлась словом, однокоренным к
  648: слову из приведенного заголовка. Назовите ИХ.
  649: 
  650: Ответ:
  651: Маски.
  652: 
  653: Комментарий:
  654: Металлическая маска под названием "личина" была частью военного шлема.
  655: 
  656: Источник:
  657:    1. "Вокруг света", 2016, N 7.
  658:    2. http://ru.wikipedia.org/wiki/Личина
  659: 
  660: Автор:
  661: Дмитрий Башук (Харьков)
  662: 
  663: Вопрос 7:
  664: Площадь ЕЕ составляет около 4152 квадратных километров и с каждым годом
  665: увеличивается. В 1991 году румынская часть, составляющая примерно 83%
  666: общей площади, была признана ЮНЕСКО объектом Всемирного природного
  667: наследия. Назовите ЕЕ двумя словами, которые начинаются на одну и ту же
  668: букву.
  669: 
  670: Ответ:
  671: Дельта Дуная.
  672: 
  673: Комментарий:
  674: Остальные 17% принадлежат Украине; с каждым годом береговая линия
  675: Черного моря смещается метров на сорок вглубь моря из-за наносов.
  676: 
  677: Источник:
  678: http://ru.wikipedia.org/wiki/Дельта_Дуная
  679: 
  680: Автор:
  681: Александр Лисянский (Харьков)
  682: 
  683: Вопрос 8:
  684: В отличие от математики, в жизни минус на минус не всегда дает плюс.
  685: Видимо, поэтому, согласно закону города Лик-Спрингс в штате Индиана,
  686: выпуская на улицу черных котов в определенный день, хозяева обязаны
  687: надевать на своих питомцев колокольчики. Назовите этот день абсолютно
  688: точно.
  689: 
  690: Ответ:
  691: Пятница, 13-е.
  692: 
  693: Комментарий:
  694: Авторы закона, видимо, считают, что если нельзя избежать негативного
  695: влияния пятницы, 13-го, то хотя бы черных котов, собирающихся перейти
  696: дорогу, горожане в этот день заблаговременно обнаружат по звуку
  697: колокольчиков. :-)
  698: 
  699: Источник:
  700: http://www.koshsps.ru/blackcat.php
  701: 
  702: Автор:
  703: Дмитрий Башук (Харьков)
  704: 
  705: Вопрос 9:
  706: Персонаж книги Хольма Ван Зайчика рассуждает: "Невозможно представить
  707: себе в просторах Запретного города, скажем, велосипед... это вопиюще
  708: несообразно и даже несколько оскорбительно - как если бы, скажем, ИКС
  709: вдруг запел...". Чуть больше сорока пяти лет назад так и произошло.
  710: Назовите ИКСА.
  711: 
  712: Ответ:
  713: Иисус Христос.
  714: 
  715: Комментарий:
  716: В 1970 году была создана рок-опера "Иисус Христос - суперзвезда".
  717: 
  718: Источник:
  719:    1. Х. Ван Зайчик. Дело Судьи Ди. http://www.flibusta.is/b/73894/read
  720:    2. http://ru.wikipedia.org/wiki/Иисус_Христос_%E2%80%94_суперзвезда
  721: 
  722: Автор:
  723: Дмитрий Башук (Харьков)
  724: 
  725: Вопрос 10:
  726: Одной из причин распада этой группы в апреле 1981 года Лоуренс ДжУбер
  727: считает то, что ее лидер стал бояться концертов (цитата): "Ведь ему
  728: приходилось бы каждые десять минут вздрагивать, ожидая, что какой-нибудь
  729: придурок выстрелит в него из пистолета". Назовите эту группу.
  730: 
  731: Ответ:
  732: "Wings".
  733: 
  734: Комментарий:
  735: Джубер - гитарист этой группы. Убийство Джона Леннона, произошедшее 8
  736: декабря 1980 года, сильно потрясло Пола Маккартни, создавшего "Wings" в
  737: 1971 году, вскоре после распада "Битлз".
  738: 
  739: Источник:
  740: http://ru.wikipedia.org/wiki/Wings
  741: 
  742: Автор:
  743: Дмитрий Башук (Харьков)
  744: 
  745: Вопрос 11:
  746: На западе США колючую проволоку часто использовали в качестве АЛЬФЫ,
  747: чтобы не беспокоиться при возможных порывах ветра. Назовите АЛЬФУ двумя
  748: словами.
  749: 
  750: Ответ:
  751: Бельевая веревка.
  752: 
  753: Комментарий:
  754: Шипы колючей проволоки легко удерживали сохнущую на ветру одежду.
  755: 
  756: Источник:
  757: http://warspot.ru/2560-shipy-bez-roz
  758: 
  759: Автор:
  760: Максим Евланов (Харьков)
  761: 
  762: Вопрос 12:
  763: "Ты и есть мой дом" - такова, по мнению персонажа Макса Фрая, идеальная
  764: ОНА. Премьера "ЕЕ" состоялась 30 декабря 1984 года. Назовите ЕЕ.
  765: 
  766: Ответ:
  767: Формула любви.
  768: 
  769: Комментарий:
  770: Во втором случае речь идет о фильме Марка Захарова.
  771: 
  772: Источник:
  773:    1. М. Фрай. Сказки старого Вильнюса II.
  774: http://www.flibusta.is/b/431642/read
  775:    2. http://ru.wikipedia.org/wiki/Формула_любви
  776: 
  777: Автор:
  778: Дмитрий Башук (Харьков)
  779: 
  780: Тур:
  781: Второй игровой день. 1 тур
  782: 
  783: Дата:
  784: 11-Nov-2016
  785: 
  786: Редактор:
  787: Артем Матухно (Одесса)
  788: 
  789: Инфо:
  790: Редактор благодарит за помощь и дельные советы во время тестирования:
  791: Дениса Гончара, Андрея Дунаева, Ирину Буртненко, Игоря Гайдута, Сергея
  792: Гайкова, Елену Григоращенко, Андрея Герасименко, Евгения Криворученко,
  793: Алексея Топышева, Андрея Ландера, Влада Гаврилова.
  794: 
  795: Вопрос 1:
  796: Для одной традиции компания "Гиннесс" использует вместо ЭТОГО продукцию
  797: собственного производства. В США в годы "сухого закона" вместо ЭТОГО
  798: использовали кока-колу. Назовите ЭТО.
  799: 
  800: Ответ:
  801: Шампанское.
  802: 
  803: Зачет:
  804: Бутылка шампанского.
  805: 
  806: Комментарий:
  807: Речь идет о традиции "крещения корабля" с помощью бутылки шампанского.
  808: Пивоваренная фирма "Гиннесс" использует при крещении своих судов,
  809: предназначенных для перевозки пива, собственное пиво. В добрый путь,
  810: друзья!
  811: 
  812: Источник:
  813: http://www.rg-rb.de/index.php?option=com_rg&task=item&id=15792&Itemid=0
  814: 
  815: Автор:
  816: Артем Матухно (Одесса)
  817: 
  818: Вопрос 2:
  819: В 1934 году подготовленная тренером КатрИн Кёртис группа из шестидесяти
  820: человек под названием "Современные русалки" с успехом выступила на
  821: Всемирной выставке в Чикаго. Какое словосочетание из двух слов впервые в
  822: истории использовал диктор НОрман Росс, комментируя это выступление?
  823: 
  824: Ответ:
  825: Синхронное плавание.
  826: 
  827: Комментарий:
  828: Правда, сама Катрин предпочитала термин "ритмическое плавание".
  829: 
  830: Источник:
  831: David Goldblatt. How to Watch the Olympics: An Instant Initiation into
  832: Every Sport at Rio-2016.
  833: https://books.google.ru/books?id=RqaCCwAAQBAJ&pg=PT366#v=onepage&q&f=false
  834: 
  835: Автор:
  836: Артем Матухно (Одесса)
  837: 
  838: Вопрос 3:
  839: [Ведущему: начинать читать текст вопроса со слова "внимание" - это не
  840: ошибка и не опечатка.]
  841:    Внимание, вопрос номер два!
  842:    Рассказывают, что некоторое время ОН, уроженец Екатеринославской
  843: губернии, специально посещал школьные уроки, где, сидя с блокнотом на
  844: задних партах, что-то зарисовывал. Назовите ЕГО.
  845: 
  846: Ответ:
  847: [Федор Павлович] Решетников.
  848: 
  849: Комментарий:
  850: Художник посещал уроки в школе перед написанием своей известной картины
  851: "Опять двойка". Ведущий намеренно ошибся, чуть не сделав третий вопрос
  852: "опять вторым".
  853: 
  854: Источник:
  855: http://www.vokrugsveta.ru/article/252015/
  856: 
  857: Автор:
  858: Артем Матухно (Одесса)
  859: 
  860: Вопрос 4:
  861: Военачальника Николая Муравьёва-Карсского и декабриста Петра Колошина
  862: объединяла настолько близкая дружба, что даже своих коней они назвали в
  863: честь ИКСОВ. В честь ИКСОВ названы две соседние вершины-четырехтысячники
  864: в Альпах. Назовите имена ИКСОВ.
  865: 
  866: Ответ:
  867: Кастор, Поллукс.
  868: 
  869: Зачет:
  870: Кастор, Полидевк.
  871: 
  872: Комментарий:
  873: ИКСЫ - Диоскуры. Муравьёв-Карсский и Колошин были настолько близкими
  874: друзьями, что часто называли друг друга братьями. Мифические герои,
  875: близнецы Кастор и Поллукс, были известны своей неразлучной дружбой.
  876: 
  877: Источник:
  878:    1. Н.Н. Муравьёв-Карсский. Собственные записки. 1811-1816.
  879: http://www.flibusta.is/b/450099/read
  880:    2. http://en.wikipedia.org/wiki/Castor_(mountain)
  881:    3. http://en.wikipedia.org/wiki/Pollux_(mountain)
  882: 
  883: Автор:
  884: Артем Матухно (Одесса)
  885: 
  886: Вопрос 5:
  887: По словам Юрия Рылёва, автора книги "6000 изобретений XX и XXI веков,
  888: изменивших мир", разработка противозачаточных таблеток в середине
  889: прошлого века стала залпом "Авроры" для НЕЕ. Назовите ЕЕ точно.
  890: 
  891: Ответ:
  892: Сексуальная революция.
  893: 
  894: Источник:
  895: Ю.И. Рылёв. 6000 изобретений XX и XXI веков, изменившие мир.
  896: http://www.flibusta.is/b/377039/read
  897: 
  898: Автор:
  899: Артем Матухно (Одесса)
  900: 
  901: Вопрос 6:
  902: Название одного магазина, предоставляющего услуги по ремонту смартфонов,
  903: представляет собой английское слово-неологизм. Какой фрукт присутствует
  904: на логотипе этого магазина?
  905: 
  906: Ответ:
  907: Груша.
  908: 
  909: Зачет:
  910: Pear.
  911: 
  912: Комментарий:
  913: Магазин называется "re:Pear" [ре пЭа], что созвучно английскому глаголу
  914: "repair" [репЭа] - чинить, ремонтировать.
  915: 
  916: Источник:
  917: Личные наблюдения автора вопроса в Стокгольме.
  918: 
  919: Автор:
  920: Артем Матухно (Одесса)
  921: 
  922: Вопрос 7:
  923: Назовите многократного олимпийского чемпиона, чья фамилия, по одной из
  924: версий, могла быть образована от прозвища, которым когда-то называли
  925: слишком прилипчивого зануду.
  926: 
  927: Ответ:
  928: [Евгений Викторович] Плющенко.
  929: 
  930: Комментарий:
  931: Прилипчивый, словно плющ. Евгений Плющенко - многократный олимпийский
  932: чемпион в фигурном катании.
  933: 
  934: Источник:
  935:    1. Т.Ф. Ведина. Энциклопедия русских фамилий. Тайны происхождения и
  936: значения. http://www.flibusta.is/b/154189/read
  937:    2. http://ru.wikipedia.org/wiki/Плющенко,_Евгений_Викторович
  938: 
  939: Автор:
  940: Артем Матухно (Одесса)
  941: 
  942: Вопрос 8:
  943: [Ведущему: при чтении выделить интонацией словосочетание "не он".]
  944:    Неизвестно, нравилось ли это произведение издателю, но в изначальном
  945: авторском варианте был не ОН, а "ключ". В каком южном городе ОН
  946: находится до сих пор?
  947: 
  948: Ответ:
  949: Бахчисарай.
  950: 
  951: Комментарий:
  952: Речь идет о Бахчисарайском фонтане. Пушкин изначально назвал свою поэму
  953: "Бахчисарайский ключ", а в "фонтан" она превратилась по воле издателя -
  954: князя Вяземского. Возможно, он считал, что авторский вариант был, как
  955: говорится, "не фонтан".
  956: 
  957: Источник:
  958: http://www.vokrugsveta.ru/vs/article/8482/
  959: 
  960: Автор:
  961: Артем Матухно (Одесса)
  962: 
  963: Вопрос 9:
  964: Согласно "Словарю морского жаргона", АЛЬФА - это прозвище
  965: "привилегированной женской особы, работающей в команде судна". АЛЬФОЙ
  966: была персонаж серии детских книг, написанных в середине прошлого века.
  967: Назовите АЛЬФУ двумя словами.
  968: 
  969: Ответ:
  970: Капитанская дочка.
  971: 
  972: Комментарий:
  973: АЛЬФА - капитанская дочка. Пеппи была дочкой капитана Эфраима
  974: Длинныйчулок.
  975: 
  976: Источник:
  977:    1. Н.А. Каланов. Словарь морского жаргона. - М.: Моркнига, 2010.
  978:    2. http://ru.wikipedia.org/wiki/Пеппи_Длинныйчулок
  979: 
  980: Автор:
  981: Артем Матухно (Одесса)
  982: 
  983: Вопрос 10:
  984: В одном кроссворде слово "рикша" определяется как "таксист, которого
  985: [ДВА СЛОВА ПРОПУЩЕНО]". А кого [ДВА СЛОВА ПРОПУЩЕНО], согласно русской
  986: пословице?
  987: 
  988: Ответ:
  989: Волка.
  990: 
  991: Комментарий:
  992: Пропущены слова "ноги кормят". Пословица - "Волка ноги кормят".
  993: 
  994: Источник:
  995:    1. http://www.kotvet.ru/odn/145
  996:    2. http://slovarick.ru/205/
  997: 
  998: Автор:
  999: Артем Матухно (Одесса)
 1000: 
 1001: Вопрос 11:
 1002: По одной из версий, ИКС - это переиначенное на английский лад выражение,
 1003: с помощью которого французские портовые грузчики просили коллег о
 1004: помощи. Какое слово мы заменили словом "ИКС"?
 1005: 
 1006: Ответ:
 1007: Mayday.
 1008: 
 1009: Зачет:
 1010: Мэйдэй; мэдэ.
 1011: 
 1012: Комментарий:
 1013: Речь идет о сигнале помощи "Mayday" [мэйдэй], который представляет собой
 1014: искаженное французское "m'aide" [мэд] - "помоги мне".
 1015: 
 1016: Источник:
 1017: Ю.И. Рылёв. 6000 изобретений XX и XXI веков, изменившие мир.
 1018: http://www.flibusta.is/b/377039/read
 1019: 
 1020: Автор:
 1021: Артем Матухно (Одесса)
 1022: 
 1023: Вопрос 12:
 1024: Рассказывают, что перед съемками "Терминатора" Арнольд Шварценеггер
 1025: специально подолгу проводил время на стрельбище, дабы научиться спускать
 1026: курок, не ДЕЛАЯ ЭТО. Как называется игра, в которой нельзя ДЕЛАТЬ ЭТО?
 1027: 
 1028: Ответ:
 1029: Гляделки.
 1030: 
 1031: Комментарий:
 1032: ДЕЛАТЬ ЭТО - моргать. Шварценеггер полагал, что роботы не моргают.
 1033: 
 1034: Источник:
 1035:    1. https://tjournal.ru/p/terminator-30-years
 1036:    2. http://ru.wiktionary.org/wiki/гляделки
 1037: 
 1038: Автор:
 1039: Артем Матухно (Одесса)
 1040: 
 1041: Тур:
 1042: Второй игровой день. 2 тур
 1043: 
 1044: Дата:
 1045: 11-Nov-2016
 1046: 
 1047: Редактор:
 1048: Дмитрий Петров и Михаил Локшин (Санкт-Петербург)
 1049: 
 1050: Инфо:
 1051: Редакторы благодарят за тестирование вопросов и ценные замечания:
 1052: Алексея и Марию Трефиловых, Артема Корсуна, Владимира Салия, Кристину
 1053: Кораблину, Александра Мудрого, Сергея Терентьева, Сергея Лобачёва,
 1054: Ярослава Косырева, команду Павла Ершова и лично Павла Ершова, а также
 1055: команды "Кадис.ру", "Ноев ковчег" и "Постмодернистское название".
 1056: 
 1057: Вопрос 1:
 1058: В Крыму первый ОН появился благодаря Максимилиану Волошину. По мнению
 1059: Сергея Федина, по уму встречают только на НЕМ. Назовите ЕГО двумя
 1060: словами.
 1061: 
 1062: Ответ:
 1063: Нудистский пляж.
 1064: 
 1065: Комментарий:
 1066: Там по одежке не встречают. Волошину принадлежат труды "Блики. Нагота" и
 1067: "Блики. Маски. Нагота". И даже Ленин считал, что в движении нудистов
 1068: есть "здоровое пролетарское начало".
 1069: 
 1070: Источник:
 1071:    1. http://ru.wikipedia.org/wiki/Натуризм
 1072:    2. http://www.gramma.ru/RST/?id=3.810
 1073: 
 1074: Автор:
 1075: Михаил Локшин (Санкт-Петербург)
 1076: 
 1077: Вопрос 2:
 1078: Сахарную голову можно поджечь, если с ней СДЕЛАТЬ ЭТО. Впрочем, порой
 1079: ЭТО ДЕЛАЮТ не только с сахарной. Какие два слова на одну и ту же букву
 1080: мы заменили словами "СДЕЛАТЬ ЭТО"?
 1081: 
 1082: Ответ:
 1083: Посыпать пеплом.
 1084: 
 1085: Комментарий:
 1086: Пепел (в частности, табачный) содержит соли калия, в том числе поташ,
 1087: катализирующие горение сахарозы.
 1088: 
 1089: Источник:
 1090:    1. http://www.youtube.com/watch?v=bckN7iMhjmg
 1091:    2. http://p-i-f.livejournal.com/6982595.html
 1092: 
 1093: Автор:
 1094: Михаил Локшин (Санкт-Петербург)
 1095: 
 1096: Вопрос 3:
 1097: Виктор Гюго провел несколько лет в изгнании на скалистом острове Гернси.
 1098: Бодлер насмешливо писал, что терзаемый лишь уколами тщеславия Гюго любил
 1099: воображать себя... Кем?
 1100: 
 1101: Ответ:
 1102: Прометеем.
 1103: 
 1104: Комментарий:
 1105: Виктор Гюго, не принявший власть Наполеона III, был своего рода
 1106: бунтарем, как и Прометей. А Бодлер был весьма циничен и товарищей по
 1107: перу недолюбливал.
 1108: 
 1109: Источник:
 1110: А. Труайя. Бодлер. http://www.flibusta.is/b/291671/read
 1111: 
 1112: Автор:
 1113: Дмитрий Петров (Санкт-Петербург)
 1114: 
 1115: Вопрос 4:
 1116: Живший в XVI веке римский папа Сикст V любил грандиозные проекты. Так,
 1117: папа обещал оплатить ЕЕ создание, если ОНА доберется до цели. Для НЕЕ
 1118: была разработана специальная система сигналов. Назовите ЕЕ.
 1119: 
 1120: Ответ:
 1121: Непобедимая армада.
 1122: 
 1123: Зачет:
 1124: Испанская армада; Великая армада.
 1125: 
 1126: Комментарий:
 1127: Сикст V обещал оплатить создание армады после высадки испанцев в Англии.
 1128: В состав флота входили корабли нескольких государств. Специально для
 1129: Армады был создан интернациональный свод морских сигналов.
 1130: 
 1131: Источник:
 1132:    1. Р. Хьюз. Рим. История города: его культура, облик, люди. - М.:
 1133: АСТ: CORPUS, 2014. - С. 288.
 1134:    2. http://ru.wikipedia.org/wiki/Непобедимая_армада
 1135: 
 1136: Автор:
 1137: Дмитрий Петров (Санкт-Петербург)
 1138: 
 1139: Вопрос 5:
 1140: Внимание, в вопросе есть замена.
 1141:    Премьер-министр Турции на полном серьезе заявил: взгляды турецких
 1142: мусульман отличаются от взглядов боевиков ИГИЛ на целых ДВА ИКС.
 1143: Ответьте точно, что мы заменили словами "ДВА ИКС".
 1144: 
 1145: Ответ:
 1146: 360 градусов.
 1147: 
 1148: Комментарий:
 1149: Ахмет Давутоглу сказал, что взгляды турецких мусульман отличаются от
 1150: взглядов террористов не на 180, а на целых 360 градусов. На всякий
 1151: случай упоминаем, что террористическая организация "Исламское
 1152: Государство" запрещена на территории РФ.
 1153: 
 1154: Источник:
 1155: http://www.middleeasteye.net/news/turkish-pm-mocked-over-360-degrees-difference-slip-190994676
 1156: 
 1157: Автор:
 1158: Дмитрий Петров (Санкт-Петербург)
 1159: 
 1160: Вопрос 6:
 1161: Пикап Toyota Hilux [тойОта хАйлакс] надежен и прост в ремонте. По словам
 1162: одного американского военнослужащего, Toyota Hilux стал автомобильным
 1163: эквивалентом ЕГО. Назовите ЕГО двумя словами.
 1164: 
 1165: Ответ:
 1166: Автомат Калашникова.
 1167: 
 1168: Комментарий:
 1169: Toyota Hilux из-за технических характеристик чаще всего используется как
 1170: импровизированная боевая машина в локальных конфликтах.
 1171: 
 1172: Источник:
 1173:    1. http://www.businessinsider.com/why-isis-uses-toyota-trucks-2015-10
 1174:    2. http://ru.wikipedia.org/wiki/Toyota_Hilux
 1175: 
 1176: Автор:
 1177: Дмитрий Петров (Санкт-Петербург)
 1178: 
 1179: Вопрос 7:
 1180: На фабрике ЕГО отца делали детали к самолетам "Мицубиси". Герой ЕГО
 1181: произведения говорит: "Самолет - это прекрасная мечта, а конструктор -
 1182: тот, кто ее воплощает". Назовите ЕГО.
 1183: 
 1184: Ответ:
 1185: [Хаяо] Миядзаки.
 1186: 
 1187: Комментарий:
 1188: Разнообразные летательные аппараты, в первую очередь самолеты, можно
 1189: увидеть во множестве его произведений: "Небесный замок Лапута", "Ветер
 1190: крепчает", "Навсикая из Долины Ветров", "Порко Россо" и других. Фразу из
 1191: вопроса произносит авиаконструктор из фильма "Ветер крепчает".
 1192: 
 1193: Источник:
 1194:    1. http://otium.su/xayao-miadzaki-velikij-fantazyor/
 1195:    2. http://ru.wikipedia.org/wiki/Миядзаки,_Хаяо
 1196:    3. Мультфильм "Ветер крепчает" (2013), реж. Хаяо Миядзаки, 12-я
 1197: минута. http://ru.wiki2.org/wiki/Ветер_крепчает?s=The%20Wind%20Rises
 1198: 
 1199: Автор:
 1200: Михаил Локшин (Санкт-Петербург)
 1201: 
 1202: Вопрос 8:
 1203: Разработанный Майком Томпсоном прибор для поиска сигнала Wi-Fi выглядит
 1204: как ОНА. Считается, что название деревни Аргури близ горы Арарат связано
 1205: с НЕЙ. Назовите ЕЕ одним словом.
 1206: 
 1207: Ответ:
 1208: Лоза.
 1209: 
 1210: Комментарий:
 1211: Дизайнер решил, что поиски сигнала Wi-Fi подобны поискам воды
 1212: лозоходцами. Как известно, на горе Арарат зреет сладкий виноград.
 1213: 
 1214: Источник:
 1215:    1. http://www.etoday.ru/2011/10/ustroystvo-dlya-poiska-wifi--.php
 1216:    2. http://ru.wikipedia.org/wiki/Лозоходство
 1217:    3. http://ru.wikipedia.org/wiki/Арарат
 1218:    4. http://ru.wikisource.org/wiki/ЭСБЕ/Аргури
 1219: 
 1220: Автор:
 1221: Дмитрий Петров (Санкт-Петербург)
 1222: 
 1223: Вопрос 9:
 1224: Механизмы работы иммунитета изучены недостаточно. Ученый Александр
 1225: ХаджИдис говорит, что иммунитет - это тонкая и пока еще ТАКАЯ ОНА.
 1226: Назовите ТАКУЮ ЕЕ.
 1227: 
 1228: Ответ:
 1229: Темная материя.
 1230: 
 1231: Комментарий:
 1232: Темная материя - гипотетическая форма материи, которая не испускает
 1233: электромагнитного излучения и напрямую не взаимодействует с ним. По
 1234: словам ученого, "иммунитет - это тонкая и пока еще темная материя".
 1235: 
 1236: Источник:
 1237:    1. http://doctorpiter.ru/articles/11136/
 1238:    2. http://ru.wikipedia.org/wiki/Тёмная_материя
 1239: 
 1240: Автор:
 1241: Михаил Локшин (Санкт-Петербург)
 1242: 
 1243: Вопрос 10:
 1244: Как-то раз сотрудники одного английского музея пришли на работу и
 1245: обнаружили, что ОНА XIX века до сих пор действует. Назовите ЕЕ.
 1246: 
 1247: Ответ:
 1248: Мышеловка.
 1249: 
 1250: Комментарий:
 1251: Несмотря на отсутствие приманки, музейная мышь забралась в старинную
 1252: мышеловку и не смогла оттуда выбраться.
 1253: 
 1254: Источник:
 1255: http://blogs.reading.ac.uk/merl/2016/02/03/155-year-old-mouse-trap-claims-its-latest-victim/
 1256: 
 1257: Автор:
 1258: Дмитрий Петров (Санкт-Петербург)
 1259: 
 1260: Вопрос 11:
 1261: Единственное слово, заимствованное английским языком из швейцарского
 1262: ретороманского, обозначает ЕЕ. Один из методов предсказания ИХ тоже
 1263: происходит из Швейцарии. Назовите ЕЕ.
 1264: 
 1265: Ответ:
 1266: Лавина.
 1267: 
 1268: Комментарий:
 1269: Английское слово "avalanche" [Эвэланш] - "лавина" - пришло из
 1270: швейцарского ретороманского через французский. Один из методов оценки
 1271: устойчивости снежно-ледового покрова был разработан в швейцарской армии.
 1272: 
 1273: Источник:
 1274:    1. Г. Доррен. Лингво. Языковой пейзаж Европы.
 1275: http://www.flibusta.is/b/446640/read
 1276:    2. http://ru.wikipedia.org/wiki/Лавина
 1277: 
 1278: Автор:
 1279: Дмитрий Петров (Санкт-Петербург)
 1280: 
 1281: Вопрос 12:
 1282: Древние китайцы верили, что ракУшки каУри приносят удачу, и придали
 1283: ЭТОМУ форму ракУшки каУри. Если вы решите приготовить ЭТО, лучше
 1284: использовать лазерный принтер - иначе будет трудно что-либо разобрать.
 1285: Назовите ЭТО тремя словами.
 1286: 
 1287: Ответ:
 1288: Печенье с предсказанием.
 1289: 
 1290: Зачет:
 1291: Печенье с предсказаниями.
 1292: 
 1293: Комментарий:
 1294: Выпекая из муки съедобное подобие каури, китайцы верили, что это
 1295: принесет им удачу. Если использовать струйный принтер, шариковую или
 1296: гелевую ручку, при высокой температуре текст на бумаге может смазаться.
 1297: 
 1298: Источник:
 1299: http://www.orientalica.com/kitayskoe-pechene-s-predskazaniyami-retseptyi-i-istoriya
 1300: 
 1301: Автор:
 1302: Михаил Локшин (Санкт-Петербург)
 1303: 
 1304: Тур:
 1305: Второй игровой день. 3 тур
 1306: 
 1307: Дата:
 1308: 11-Nov-2016
 1309: 
 1310: Редактор:
 1311: Александр Мудрый (Черновцы)
 1312: 
 1313: Инфо:
 1314: Редактор благодарит за тестирование и ценные замечания: Ростислава
 1315: Гимчинского, Владимира Городецкого, Александра Зинченко, Ирину Зубкову,
 1316: Андрея Кокуленко, Николая Константинова, Артема Корсуна, Александра
 1317: Кудрявцева, Константина Науменко, Алексея и Марию Трефиловых, Игоря
 1318: Тюнькина, Ульяну Фабричнину, а также команду "Прометей" (Черновцы).
 1319: 
 1320: Вопрос 1:
 1321: Более четверти века в этом городе не функционировали полтора десятка
 1322: станций метрополитена. Назовите этот город.
 1323: 
 1324: Ответ:
 1325: Восточный Берлин.
 1326: 
 1327: Зачет:
 1328: Берлин.
 1329: 
 1330: Комментарий:
 1331: После сооружения Берлинской стены были закрыты семь станций на линии U6
 1332: и восемь станций на линии U8 в связи с тем, что эти линии шли из
 1333: западного сектора в западный через восточную часть. Было принято решение
 1334: не разрывать линии западного метрополитена, а только закрыть станции,
 1335: находящиеся в восточном секторе. Поезда через эти станции двигались, не
 1336: останавливаясь.
 1337: 
 1338: Источник:
 1339:    1. http://ru.wikipedia.org/wiki/Берлинский_метрополитен
 1340:    2. http://forum.tr.ru/read.php?5,174129,page=all
 1341: 
 1342: Автор:
 1343: Александр Мудрый (Черновцы)
 1344: 
 1345: Вопрос 2:
 1346: На мемориальной доске Траяну Поповичу, который был мэром Черновцов в
 1347: середине прошлого века, есть число 19600. Сергей Воронцов назвал
 1348: Поповича "буковинским ИМ". Назовите ЕГО.
 1349: 
 1350: Ответ:
 1351: [Оскар] Шиндлер.
 1352: 
 1353: Комментарий:
 1354: В 1941 году Попович отказался выполнить приказ о депортации черновицких
 1355: евреев в лагеря Транснистрии. Мало того, он убедил вышестоящие власти в
 1356: своей правоте и добился изменения решения: 19600 черновицких евреев
 1357: остались в городе.
 1358: 
 1359: Источник:
 1360:    1. http://gazeta.zn.ua/SOCIETY/bukovinskiy_shindler.html
 1361:    2. https://pogliad.ua/news/chernivtsi/u-chernivcyah-vidkrili-memorialnu-doshku-trayanu-popovichu-280684
 1362: 
 1363: Автор:
 1364: Александр Мудрый (Черновцы)
 1365: 
 1366: Вопрос 3:
 1367: Большинство украинских телеканалов транслируются со спутников "Amos"
 1368: [Эймос], "Sirius" [сИриус] и "HotBird" [хотбёд]. Поэтому самый
 1369: распространенный на Украине тип спутниковых антенн получил сказочное
 1370: прозвище "он". В одном из слов предыдущего предложения мы пропустили
 1371: пять букв. Напишите это слово в исходном виде.
 1372: 
 1373: Ответ:
 1374: Горыныч.
 1375: 
 1376: Комментарий:
 1377: Чтобы поймать все три спутника, на тарелку устанавливают три конвертера,
 1378: которые в просторечии называют головками. Такая "трехголовая" антенна и
 1379: получила название в честь Змея из русских сказок.
 1380: 
 1381: Источник:
 1382:    1. http://www.satsis.info/forum (поиск по слову "горыныч")
 1383:    2. http://www.satsputnik.ru/vybrat-oborudovanie-dlya-prosmotra-besplatnogo-sputnikovogo-televideniya/
 1384: 
 1385: Автор:
 1386: Александр Мудрый (Черновцы)
 1387: 
 1388: Вопрос 4:
 1389: В первом тайме матча Евро-2016 Италия - Германия обе команды практически
 1390: не предпринимали атакующих действий. Денис Казанский назвал этот тайм
 1391: "ПРОПУСК ГарОнне". Заполните пропуск двумя или тремя словами.
 1392: 
 1393: Ответ:
 1394: Стояние на реке.
 1395: 
 1396: Зачет:
 1397: Стоянием на реке; стояние на; стоянием на.
 1398: 
 1399: Комментарий:
 1400: Матч проходил в Бордо, расположенном на реке Гаронне, и комментатор
 1401: сравнил такой футбол с известными событиями российской истории.
 1402: 
 1403: Источник:
 1404: Трансляция четвертьфинала Евро-2016 Италия - Германия на канале "Матч!
 1405: Футбол 1", эфир от 02.07.2016 г.
 1406: 
 1407: Автор:
 1408: Александр Мудрый (Черновцы)
 1409: 
 1410: Вопрос 5:
 1411: Одна из версий возникновения ЭТОГО - сбой в одном из отделов мозга. В
 1412: такой момент новая информация поступает в подкорковые ядра, отвечающие
 1413: за ее переработку, не напрямую, а через другую зону. Назовите ЭТО.
 1414: 
 1415: Ответ:
 1416: Дежавю.
 1417: 
 1418: Комментарий:
 1419: Из-за сбоя в парагиппокампальной извилине новая информация поступает в
 1420: подкорковые ядра через правую височную долю, отвечающую за средне- и
 1421: долговременную память. Поэтому человек воспринимает эту новую информацию
 1422: как уже виденную.
 1423: 
 1424: Источник:
 1425: http://pikabu.ru/story/prostoe_i_ponyatnoe_obyasnenie_pochemu_myi_ispyityivaem_dezhavyu_4484414
 1426: 
 1427: Автор:
 1428: Ростислав Гимчинский (Черновцы)
 1429: 
 1430: Вопрос 6:
 1431: После того как в болотах графства Кембриджшир было обнаружено деревянное
 1432: колесо, эту территорию прозвали местными ИМИ. Назовите ИХ одним словом.
 1433: 
 1434: Ответ:
 1435: Помпеи.
 1436: 
 1437: Комментарий:
 1438: В болотах при отсутствии доступа кислорода очень хорошо сохраняются
 1439: археологические находки. Место раскопок поселения близ Питерборо, в
 1440: котором нашли дубовое колесо возрастом 3000 лет, сами археологи назвали
 1441: самым богатым памятником бронзового века в Великобритании и сравнили с
 1442: легендарными Помпеями.
 1443: 
 1444: Источник:
 1445: https://www.theguardian.com/science/2016/feb/19/archaeologists-excavate-bronze-age-wheel-cambridgeshire
 1446: 
 1447: Автор:
 1448: Александр Мудрый (Черновцы)
 1449: 
 1450: Вопрос 7:
 1451: В одном сериале маленькая компания, разрабатывающая инновационную
 1452: технологию, узнаёт, что у нее появился серьезный конкурент. Один из
 1453: персонажей с грустью говорит, что их шансы опередить этого конкурента
 1454: ЧРЕЗВЫЧАЙНО МАЛЫ. Какое соотношение мы заменили словами "ЧРЕЗВЫЧАЙНО
 1455: МАЛЫ"?
 1456: 
 1457: Ответ:
 1458: Один к гуглу.
 1459: 
 1460: Зачет:
 1461: 1:10^100.
 1462: 
 1463: Комментарий:
 1464: Конкурентом этой компании был "Google". Персонаж скаламбурил, понимая,
 1465: что шансы небольшого стартапа обогнать IT-гиганта очень невелики.
 1466: 
 1467: Источник:
 1468: Телесериал "Два с половиной человека", s11e16.
 1469: 
 1470: Автор:
 1471: Александр Мудрый (Черновцы)
 1472: 
 1473: Вопрос 8:
 1474: Внимание, ИКС в вопросе - замена.
 1475:    Получивший образование в Париже Тадеуш Костюшко устроился ИКСОМ,
 1476: чтобы быть поближе к любимой девушке. А заглавный персонаж известного
 1477: произведения стал ИКСОМ, чтобы быть поближе к своему врагу. Назовите
 1478: фамилию этого персонажа.
 1479: 
 1480: Ответ:
 1481: Дубровский.
 1482: 
 1483: Комментарий:
 1484: ИКС - гувернёр, учитель французского. Костюшко устроился гувернёром в
 1485: дом богатого помещика Сосновского, в дочь которого Людвику был влюблен.
 1486: 
 1487: Источник:
 1488:    1. Документальный фильм "Тадеуш Костюшко - польский генерал".
 1489:    2. http://ru.wikipedia.org/wiki/Дубровский_(роман)
 1490: 
 1491: Автор:
 1492: Александр Мудрый (Черновцы)
 1493: 
 1494: Вопрос 9:
 1495: Скоростной хоккеист Виталий Абрамов в прошлом сезоне ярко ворвался в
 1496: одну из юниорских лиг. Павел Климовицкий назвал Абрамова ИМ. Сам Абрамов
 1497: видел ЕГО, будучи в школе. Назовите ЕГО двумя словами.
 1498: 
 1499: Ответ:
 1500: Челябинский метеор.
 1501: 
 1502: Зачет:
 1503: Челябинский метеорит.
 1504: 
 1505: Комментарий:
 1506: Абрамов был признан лучшим новичком Главной юниорской хоккейной лиги
 1507: Квебека (QMJHL). Поскольку Виталий из Челябинска, то его не преминули
 1508: сравнить с метеоритом, пролетевшим над городом утром 15 февраля 2013
 1509: года. В это время 14-летний Абрамов, как и положено, находился в школе.
 1510: 
 1511: Источник:
 1512:    1. http://www.sport-express.ru/hockey/nhl/reviews/1014473/
 1513:    2. http://www.nhl.com/ice/ru/news.htm?id=886932
 1514: 
 1515: Автор:
 1516: Александр Мудрый (Черновцы)
 1517: 
 1518: Вопрос 10:
 1519: Наблюдение за одной экзопланетой поставило под сомнение эффективность
 1520: процесса циркуляризации. Заметка об этом называлась "ЧУДАКОВАТАЯ
 1521: экзопланета задает вопросы". Какое слово мы заменили словом
 1522: "ЧУДАКОВАТАЯ"?
 1523: 
 1524: Ответ:
 1525: Эксцентричная.
 1526: 
 1527: Зачет:
 1528: Эксцентрическая.
 1529: 
 1530: Комментарий:
 1531: Эта планета расположена в созвездии Большой Медведицы и представляет
 1532: собой газовый гигант размером с Юпитер. Она вращается вокруг своей
 1533: звезды по очень вытянутой, т.е. эксцентричной орбите. Процесс
 1534: циркуляризации состоит в "выравнивании" орбиты планеты, постепенном
 1535: приближении ее к круговой. Но в данном случае он происходит гораздо
 1536: медленнее, чем предсказывает теория.
 1537: 
 1538: Источник:
 1539:    1. http://www.nkj.ru/news/28463/
 1540:    2. http://dic.academic.ru/dic.nsf/dic_synonims/201591/
 1541: 
 1542: Автор:
 1543: Александр Мудрый (Черновцы)
 1544: 
 1545: Вопрос 11:
 1546: Рассказывая о заядлом рыбаке, Вашингтон Ирвинг пишет, что тот живет
 1547: жизнью настоящего ЕГО. ОНИ, вероятно, происходят от кистепёрых рыб.
 1548: Назовите ИХ двухкоренным словом.
 1549: 
 1550: Ответ:
 1551: Земноводные.
 1552: 
 1553: Комментарий:
 1554: Рыбак из рассказа Ирвинга в воде и около нее проводил не меньше времени,
 1555: чем на суше. Биологи полагают, что кистепёрые рыбы дали начало
 1556: земноводным и первыми из позвоночных вышли на сушу.
 1557: 
 1558: Источник:
 1559:    1. В. Ирвинг. Происшествие с черным рыбаком.
 1560: http://www.flibusta.is/b/70482/read
 1561:    2. http://ru.wikipedia.org/wiki/Происхождение_земноводных
 1562:    3. http://ru.wikipedia.org/wiki/Кистепёрые_рыбы
 1563: 
 1564: Автор:
 1565: Александр Мудрый (Черновцы)
 1566: 
 1567: Вопрос 12:
 1568: Назовите роман, первый перевод которого на русский язык выполнила в 1936
 1569: году Евгения Калашникова.
 1570: 
 1571: Ответ:
 1572: "Прощай, оружие!".
 1573: 
 1574: Комментарий:
 1575: Вот такое любопытное совпадение. Кстати, по мнению Корнея Чуковского,
 1576: этот перевод принадлежит к высшим достижениям советского переводческого
 1577: искусства.
 1578: 
 1579: Источник:
 1580: http://ru.wikipedia.org/wiki/Калашникова,_Евгения_Давыдовна
 1581: 
 1582: Автор:
 1583: Александр Мудрый (Черновцы)
 1584: 
 1585: Тур:
 1586: Третий игровой день. 1 тур
 1587: 
 1588: Дата:
 1589: 09-Dec-2016
 1590: 
 1591: Редактор:
 1592: Антон Волосатов и Константин Сахаров (Ивантеевка)
 1593: 
 1594: Инфо:
 1595: Помощи в подготовке уделили свое ценное время: Никита Коровин (Манила),
 1596: Дмитрий Дягилев, Татьяна Левченко, Галина Пактовская, Серафим Шибанов
 1597: (все - Москва), Александр Усков и Олег Михеев (оба - Краснодар), Николай
 1598: Коврижных (Киров), Екатерина Сахарова (Ивантеевка).
 1599: 
 1600: Вопрос 1:
 1601: [Ведущему: отточие не озвучивать.]
 1602:    Заметка о чемпионате по какой игре вышла под заголовком "... Без
 1603: покемонов"?
 1604: 
 1605: Ответ:
 1606: Го.
 1607: 
 1608: Комментарий:
 1609: Мобильная игра "Pokémon Go" крайне популярна (или была таковой
 1610: несколько месяцев назад), но и поклонников интеллектуальной игры го в
 1611: мире насчитывают до 60 миллионов. Больше вопросов о покемонах и по
 1612: заголовкам в туре не будет!
 1613: 
 1614: Источник:
 1615: https://www.championat.com/other/article-252640-v-sankt-peterburge-prohodit-60-j-evropejskij-kongress-go.html
 1616: 
 1617: Автор:
 1618: Константин Сахаров (Ивантеевка)
 1619: 
 1620: Вопрос 2:
 1621: Однажды автор вопроса с друзьями играли в "Угадай кто", и одному из
 1622: игроков достался популярный персонаж американского мультсериала.
 1623: Воспроизведите первый вопрос, который задал о себе игрок.
 1624: 
 1625: Ответ:
 1626: "Я в данный момент умер?".
 1627: 
 1628: Зачет:
 1629: По упоминанию смерти или жизни.
 1630: 
 1631: Комментарий:
 1632: Кенни из "South Park" известен тем, что постоянно умирает и воскресает,
 1633: поэтому вопрос поставил партнеров в тупик.
 1634: 
 1635: Источник:
 1636: ЛОАВ.
 1637: 
 1638: Автор:
 1639: Константин Сахаров (Ивантеевка)
 1640: 
 1641: Вопрос 3:
 1642: Американская чемпионка мира в детстве нередко терпела оскорбления на
 1643: религиозной почве. Поэтому вид спорта был выбран такой, чтобы не
 1644: испытывать проблем с экипировкой. Назовите этот вид спорта.
 1645: 
 1646: Ответ:
 1647: Фехтование.
 1648: 
 1649: Комментарий:
 1650: Одежда девушек-мусульман не должна открывать руки и ноги. В фехтовании
 1651: тело и даже лицо спортсмена полностью закрыто униформой. Так что мама
 1652: разрешила ИбтихАдж МухаммАд заниматься спортом, увидев подходящие
 1653: костюмы фехтовальщиц. В Рио-де-Жанейро Мухаммад стала первой
 1654: американкой, которая выступала на Олимпиаде в хиджабе.
 1655: 
 1656: Источник:
 1657:    1. http://www.sports.ru/tribuna/blogs/innuendo/1020658.html
 1658:    2. http://ru.wikipedia.org/wiki/Мухаммад,_Ибтихадж
 1659: 
 1660: Автор:
 1661: Константин Сахаров (Ивантеевка)
 1662: 
 1663: Вопрос 4:
 1664: Белый крест на зеленом фоне - таким предлагали сделать флаг ЕЕ. Назовите
 1665: ЕЕ.
 1666: 
 1667: Ответ:
 1668: Гренландия.
 1669: 
 1670: Комментарий:
 1671: Гренландия - зависимая территория Дании, и собственный флаг ей
 1672: предлагали сделать по аналогии со всеми скандинавскими, но с учетом
 1673: "зеленого" названия территории. Вероятно, из-за стремления подчеркнуть
 1674: самостоятельность нынешний флаг скандинавского креста не содержит.
 1675: 
 1676: Источник:
 1677: http://ru.wikipedia.org/wiki/Флаг_Гренландии
 1678: 
 1679: Автор:
 1680: Константин Сахаров (Ивантеевка)
 1681: 
 1682: Вопрос 5:
 1683: Обсуждая фильм 2015 года, научный сотрудник Артем Климчук призвал ценить
 1684: знания, потому что они могут пригодиться в самый неожиданный момент. По
 1685: его мнению, от негативных коннотаций должно избавиться слово, означающее
 1686: специалиста. Напишите это слово.
 1687: 
 1688: Ответ:
 1689: Ботаник.
 1690: 
 1691: Комментарий:
 1692: В фильме "Марсианин" главный герой - по основной профессии ботаник -
 1693: сумел применить свои знания и выжить долгое время на Марсе. "Если
 1694: понятие "ботаник" воспринимать не как человека, который занимается
 1695: растениями, а как человека, который много чего знает и хорошо учился в
 1696: институте, то краткая мораль фильма - будьте ботаниками. Это вас
 1697: спасет".
 1698: 
 1699: Источник:
 1700: http://www.the-village.ru/village/weekend/oba/223751-the-martian-mfti
 1701: 
 1702: Автор:
 1703: Константин Сахаров (Ивантеевка)
 1704: 
 1705: Вопрос 6:
 1706: Комментируя итоги выборов осенью 2016 года, Эльвира Набиуллина заявила:
 1707: "Мы, конечно, все помним про песню". Из какого города группа,
 1708: исполнившая эту песню?
 1709: 
 1710: Ответ:
 1711: Из Владивостока.
 1712: 
 1713: Комментарий:
 1714: Выбор городов для размещения на новых рублевых купюрах завершился в
 1715: пользу Севастополя и Дальнего Востока. Решение о том, какой из городов
 1716: окажется на какой купюре, будет принимать совет директоров Центробанка
 1717: России (Набиуллина - его глава). Песня, о которой идет речь, -
 1718: "Владивосток 2000" группы "Мумий Тролль".
 1719: 
 1720: Источник:
 1721: http://www.vesti.ru/doc.html?id=2809499
 1722: 
 1723: Автор:
 1724: Константин Сахаров (Ивантеевка)
 1725: 
 1726: Вопрос 7:
 1727: В Эфиопии и Эритрее практикуется ОНА, включающая в себя такие стадии:
 1728: обжарка на углях, перемалывание в деревянной ступке, варка в котле,
 1729: перелив в другой сосуд и охлаждение, просеивание через фильтр из
 1730: конского волоса и наконец, разлив в кипящем состоянии. Назовите ЕЕ двумя
 1731: словами.
 1732: 
 1733: Ответ:
 1734: Кофейная церемония.
 1735: 
 1736: Комментарий:
 1737: Мы попытались дать описание, напоминающее более известную чайную
 1738: церемонию. Упомянутые африканские регионы известны своим кофе.
 1739: 
 1740: Источник:
 1741: http://www.intensocoffee.ru/articles.php?article=3_kofeinaja_ceremonia
 1742: 
 1743: Автор:
 1744: Антон Волосатов (Ивантеевка)
 1745: 
 1746: Вопрос 8:
 1747: На логотипе фирмы "Деловой звук" изображен несуществующий вид ЕГО -
 1748: скрипичный ключ. Назовите ЕГО двумя словами.
 1749: 
 1750: Ответ:
 1751: Галстучный узел.
 1752: 
 1753: Зачет:
 1754: Узел галстука; способ завязывания.
 1755: 
 1756: Комментарий:
 1757: Галстук - пожалуй, главнейший элемент делового стиля.
 1758: 
 1759: Источник:
 1760: http://www.business-sound.ru/
 1761: 
 1762: Автор:
 1763: Антон Волосатов (Ивантеевка)
 1764: 
 1765: Вопрос 9:
 1766: "Самый великий человек в мире - великий вождь Ким Ир Сен". Как
 1767: утверждает журнал "Максим", за эту фразу любого северокорейца отправили
 1768: бы в лагерь, так как ОНО не на месте. Напишите ЕГО.
 1769: 
 1770: Ответ:
 1771: Ким Ир Сен.
 1772: 
 1773: Зачет:
 1774: Имя [вождя].
 1775: 
 1776: Комментарий:
 1777: В КНДР имя Ким Ир Сена всегда должно стоять в начале предложения. Мы
 1778: нарушили это правило уже дважды. :-(
 1779: 
 1780: Источник:
 1781: http://www.maximonline.ru/longreads/get-smart/_article/north-korea/
 1782: 
 1783: Автор:
 1784: Илья Иванов (Москва)
 1785: 
 1786: Вопрос 10:
 1787: Когда ОНИ появились в Италии в начале прошлого века, за характерный
 1788: элемент ИХ стали называть "fumetti" [фумЕтти], что означает "дымок".
 1789: Ответьте одним словом, что такое ОНИ.
 1790: 
 1791: Ответ:
 1792: Комиксы.
 1793: 
 1794: Комментарий:
 1795: Итальянцам казалось, что спичбаблы в американских комиксах похожи на
 1796: облачка дыма, так что стали именовать так и сами комиксы.
 1797: 
 1798: Источник:
 1799:    1. Д.Е. Комм. Формулы страха. Введение в историю и теорию фильма
 1800: ужасов. http://www.flibusta.is/b/353326/read
 1801:    2. http://en.wikipedia.org/wiki/Italian_comics
 1802: 
 1803: Автор:
 1804: Руслан Хаиткулов (Москва)
 1805: 
 1806: Вопрос 11:
 1807: Искусствовед Эрвин ПанОфский пришел к выводу, что крылатый мальчик -
 1808: Эрот, а его связанные руки указывают на "вынужденное целомудрие".
 1809: Назовите заглавную героиню описанной картины.
 1810: 
 1811: Ответ:
 1812: Даная.
 1813: 
 1814: Комментарий:
 1815: Как известно, Зевс вступил в отношения с Данаей не какими-то привычными
 1816: методами, а в виде золотого дождя.
 1817: 
 1818: Источник:
 1819: http://ru.wikipedia.org/wiki/Даная_(картина_Рембрандта)
 1820: 
 1821: Автор:
 1822: Константин Сахаров (Ивантеевка)
 1823: 
 1824: Вопрос 12:
 1825: С 2016 года для НЕЕ официально введено ограничение в 45 секунд. В марте
 1826: 1946 года Рэй МИлланд от НЕЕ отказался, ограничившись поклоном, что до
 1827: сих пор остается уникальным случаем. Назовите ЕЕ максимально точно.
 1828: 
 1829: Ответ:
 1830: Речь победителя на вручении премии "Оскар".
 1831: 
 1832: Зачет:
 1833: По словам "речь" и "Оскар".
 1834: 
 1835: Комментарий:
 1836: Чтобы слишком не затягивать церемонию и избежать инцидентов, выступления
 1837: победителей регламентировали. Вручение наград традиционно проводится на
 1838: границе зимы и весны. Несмотря на то что в карьере Милланда это была
 1839: первая и последняя статуэтка, он проявил необычайную скромность. А
 1840: награжден он был за лучшую мужскую роль в фильме "Потерянный уик-энд".
 1841:    z-checkdb: Ограничение в 45 секунд действует с 2010 года, см.
 1842: http://www.hollywoodreporter.com/lists/oscars-watch-10-longest-acceptance-867667/item/greer-garson-10-longest-acceptance-867635
 1843: (Евгений Рубашкин).
 1844: 
 1845: Источник:
 1846:    1. http://oscar2016.ru/news/45-seconds/
 1847:    2. http://ru.wikipedia.org/wiki/Рэй_Милланд
 1848: 
 1849: Автор:
 1850: Антон Волосатов (Ивантеевка)
 1851: 
 1852: Тур:
 1853: Третий игровой день. 2 тур
 1854: 
 1855: Дата:
 1856: 09-Dec-2016
 1857: 
 1858: Редактор:
 1859: Александр Голиков и Людмила Полякова (Харьков)
 1860: 
 1861: Вопрос 1:
 1862: Героиня Набокова, опустив после сложного разговора телефонную трубку,
 1863: прислушалась и с облегчением услышала только ЕГО. Назовите ЕГО тремя
 1864: словами, начинающимися на одну и ту же букву.
 1865: 
 1866: Ответ:
 1867: Стук своего сердца.
 1868: 
 1869: Зачет:
 1870: Стучание своего сердца; стук/стучание собственного сердца.
 1871: 
 1872: Комментарий:
 1873: Она жила в большом доме и боялась, что этот разговор могли подслушать.
 1874: Рядом никого не оказалось. Это не "так судьба стучится в двери", конечно
 1875: же, но всё же.
 1876: 
 1877: Источник:
 1878: В.В. Набоков. Защита Лужина. http://www.flibusta.is/b/385732/read
 1879: 
 1880: Автор:
 1881: Людмила Полякова (Харьков)
 1882: 
 1883: Вопрос 2:
 1884: [Ведущему: внимательно прочитать конец первого предложения, чтобы
 1885: команды четко уяснили грамматическую структуру.]
 1886:    В одном фильме пилигрим сообщает, что первым по доблести был Ричард
 1887: Львиное Сердце, а ПРОПУСК. Уже в конце апреля 2016 года стало понятно,
 1888: что, несмотря на неверие букмекеров еще осенью, ПРОПУСК - и это как
 1889: минимум. Пропуски на слух неотличимы. Заполните любой из них двумя
 1890: словами.
 1891: 
 1892: Ответ:
 1893: Лестер второй.
 1894: 
 1895: Зачет:
 1896: "Лестер" - второй.
 1897: 
 1898: Комментарий:
 1899: Вторым был благородный граф Лестер.
 1900: 
 1901: Источник:
 1902: Х/ф "Баллада о доблестном рыцаре Айвенго" (1982), реж. Сергей Тарасов,
 1903: 8-я минута.
 1904: 
 1905: Автор:
 1906: Александр Голиков (Харьков)
 1907: 
 1908: Вопрос 3:
 1909: [Ведущему: четко прочитать "в Шуне" и "в Кване".]
 1910:    В самом начале пьесы один из персонажей саркастично упоминает
 1911: случайность в Шуне и случайность в Кване. Впрочем, им все-таки больше
 1912: везет... Где?
 1913: 
 1914: Ответ:
 1915: В Сычуани.
 1916: 
 1917: Зачет:
 1918: В Сезуане.
 1919: 
 1920: Комментарий:
 1921: Боги в произведении Бертольта Брехта уже не верят, что им удастся найти
 1922: хотя бы одного доброго человека, но наконец-то нашелся хотя бы один - в
 1923: городе Сычуань.
 1924: 
 1925: Источник:
 1926: Б. Брехт. Добрый человек из Сычуани.
 1927: http://www.flibusta.is/b/469976/read
 1928: 
 1929: Автор:
 1930: Александр Голиков (Харьков)
 1931: 
 1932: Вопрос 4:
 1933: Когда брехтовские боги обсуждают водоноса, один из них обвиняет его в
 1934: мошенничестве, держа в руках кружку, из которой водонос их напоил водой.
 1935: Всё дело в НЕМ. Назовите ЕГО двумя словами, начинающимися на одну и ту
 1936: же букву.
 1937: 
 1938: Ответ:
 1939: Двойное дно.
 1940: 
 1941: Комментарий:
 1942: Водонос дал богам напиться воды из кружки с двойным дном, которое
 1943: символизирует нечестность и мошенничество.
 1944: 
 1945: Источник:
 1946: Б. Брехт. Добрый человек из Сычуани.
 1947: http://www.flibusta.is/b/469976/read
 1948: 
 1949: Автор:
 1950: Александр Голиков (Харьков)
 1951: 
 1952: Вопрос 5:
 1953: Внимание, в одном из слов вопроса мы пропустили две буквы.
 1954:    Вильфредо Парето - итальянский социолог-консерватор, который,
 1955: несмотря на свой аристократизм по происхождению, пережил семейную драму.
 1956: Преподаватель социологии, рассказывая об этом, заявил, что его жена,
 1957: Александра Бакунина, изменила ему с маркизом. Восстановите исходное
 1958: слово.
 1959: 
 1960: Ответ:
 1961: Марксизмом.
 1962: 
 1963: Комментарий:
 1964: Фамилия Александры Бакуниной явно указывает на ее марксистские и
 1965: анархистские контакты.
 1966: 
 1967: Источник:
 1968:    1. ЛНА на лекции.
 1969:    2. http://50.economicus.ru/index.php?ch=5&le=42&r=4&z=1
 1970:    3. http://ru.wikipedia.org/wiki/Парето,_Вильфредо
 1971: 
 1972: Автор:
 1973: Александр Голиков (Харьков)
 1974: 
 1975: Вопрос 6:
 1976: В мае 1949 года ОН передал свой список из 38 журналистов и писателей
 1977: спецслужбам. В этом списке были такие характеристики, как "бесчестный
 1978: карьерист", "сентиментальный симпатизант", "еврейский",
 1979: "антибританский", "хорошо нажился в СССР", "очень антибелый" и тому
 1980: подобное. Назовите ЕГО.
 1981: 
 1982: Ответ:
 1983: [Эрик] Блэр.
 1984: 
 1985: Зачет:
 1986: [Джордж] Оруэлл.
 1987: 
 1988: Комментарий:
 1989: Да, марксист по убеждениям Эрик Блэр работал на Большого Брата.
 1990: 
 1991: Источник:
 1992: http://gefter.ru/archive/15452
 1993: 
 1994: Автор:
 1995: Александр Голиков (Харьков)
 1996: 
 1997: Вопрос 7:
 1998: Согласно одному несерьезному источнику, в результате забастовки цыган
 1999: тысячи горожан остались ПРОПУСК. По мнению профессора Криса Синхи,
 2000: представители племени амондава живут ПРОПУСК, поэтому могут говорить
 2001: только об одном событии. Какие два слова на одну и ту же букву мы дважды
 2002: пропустили?
 2003: 
 2004: Ответ:
 2005: Без будущего.
 2006: 
 2007: Комментарий:
 2008: Вряд ли это племя живет в мире Оруэлла, но будущего времени в его языке
 2009: действительно нет.
 2010: 
 2011: Источник:
 2012:    1. http://www.anekdot.ru/id/256514/
 2013:    2. http://www.bbc.com/russian/international/2011/05/110520_amondawa_language_lacks_time.shtml
 2014: 
 2015: Автор:
 2016: Людмила Полякова (Харьков)
 2017: 
 2018: Вопрос 8:
 2019:    <раздатка>
 2020:    leaned
 2021:    </раздатка>
 2022:    Седьмой эпизод "Звездных войн" многие обвиняют во вторичности. В
 2023: неологизме, который газета "Вашингтон Пост" применяет по отношению к
 2024: одной из планет в этом фильме, мы убрали четыре буквы. Восстановите
 2025: исходное слово.
 2026: 
 2027: Ответ:
 2028: Alderaaned.
 2029: 
 2030: Комментарий:
 2031: Планета была уничтожена, как и Алдераан, причем как способ уничтожения,
 2032: так и кинокартинка дают основания подозревать создателей фильма во
 2033: вторичности.
 2034: 
 2035: Источник:
 2036: https://www.washingtonpost.com/news/monkey-cage/wp/2016/01/06/here-are-three-ways-that-star-wars-is-a-neoconservative-universe/
 2037: 
 2038: Автор:
 2039: Людмила Полякова (Харьков)
 2040: 
 2041: Вопрос 9:
 2042: Как утверждают журналисты, город Вольфсбург подхватывает воспаление
 2043: легких, когда корпорация "Фольксваген" ДЕЛАЕТ ЭТО. Желая показать, что
 2044: он опознал фрагменты плагиата, Джоакино Россини на концерте неоднократно
 2045: ДЕЛАЛ ЭТО. Что делал?
 2046: 
 2047: Ответ:
 2048: Снимал шляпу.
 2049: 
 2050: Комментарий:
 2051: От "Фольксвагена" сильно зависит благосостояние Вольфсбурга. А Россини
 2052: "здоровался" со "встреченными" им во время концерта коллегами.
 2053: 
 2054: Источник:
 2055:    1. "Футбол", 2015, N 66 (1352). - С. 21.
 2056:    2. А.И. Муха. Музыканты смеются. http://www.flibusta.is/b/273591/read
 2057: 
 2058: Автор:
 2059: Александр Голиков (Харьков)
 2060: 
 2061: Вопрос 10:
 2062: В книге "Алиса и Алисия" положительный персонаж Алиса Селезнёва
 2063: приезжает на Суматру за одним экземпляром, путешествует во времени и
 2064: знакомится с диктатором и тираном Алисией I. Какое слово мы пропустили в
 2065: предыдущем предложении?
 2066: 
 2067: Ответ:
 2068: Бабочки.
 2069: 
 2070: Комментарий:
 2071: Как можно понять из имени, Алисия - негативное развитие Алисы.
 2072: Получается своеобразный эффект бабочки.
 2073: 
 2074: Источник:
 2075: К. Булычёв. Алиса и Алисия. http://www.flibusta.is/b/108182/read
 2076: 
 2077: Автор:
 2078: Александр Голиков (Харьков)
 2079: 
 2080: Вопрос 11:
 2081: Известный персонаж книги Бориса Акунина при виде Тенерифе произносит,
 2082: как это ни странно для русскоязычного читателя, короткое слово. Назовите
 2083: это двусложное слово.
 2084: 
 2085: Ответ:
 2086: Яма.
 2087: 
 2088: Комментарий:
 2089: Тенерифе - это остров со знаменитым вулканом Тейде. Персонаж, Масахиро
 2090: Сибата, - японец, слуга Фандорина, и для него это "яма" (сравните с
 2091: "Фудзи-яма", "Тате-яма").
 2092: 
 2093: Источник:
 2094:    1. Б. Акунин. Планета Вода. http://www.flibusta.is/b/401666/read
 2095:    2. http://ru.wikipedia.org/wiki/Тенерифе
 2096:    3. http://ru.wikipedia.org/wiki/Фудзияма
 2097:    4. http://ru.wikipedia.org/wiki/Татеяма_(гора)
 2098: 
 2099: Автор:
 2100: Людмила Полякова (Харьков)
 2101: 
 2102: Вопрос 12:
 2103: В постмодернистской пародии на пьесу "Ричард III" Эдвард расстраивается,
 2104: что его после смерти не слушают, и упоминает о планах переезда. Ответьте
 2105: абсолютно точно: куда именно?
 2106: 
 2107: Ответ:
 2108: Эльсинор.
 2109: 
 2110: Зачет:
 2111: Замок Эльсинор.
 2112: 
 2113: Комментарий:
 2114: И, как и подобает в постмодернистском тексте, Эдвард становится тенью -
 2115: видимо, тенью отца Гамлета.
 2116: 
 2117: Источник:
 2118: http://the-mockturtle.livejournal.com/770332.html
 2119: 
 2120: Автор:
 2121: Людмила Полякова (Харьков)
 2122: 
 2123: Тур:
 2124: Третий игровой день. 3 тур
 2125: 
 2126: Дата:
 2127: 09-Dec-2016
 2128: 
 2129: Редактор:
 2130: Олег Михеев и Александр Усков (Краснодар)
 2131: 
 2132: Инфо:
 2133: Редакторы благодарят за тестирование вопросов и ценные советы: Андрея
 2134: Баландина и Андрея Гречишникова (оба - Москва), Максима Карачуна
 2135: (Краснодар), Маргариту Кирюшину и Инессу Кличманову (обе - Москва), Илью
 2136: Кукушкина и Дениса Лагутина (оба - Краснодар), Наталью Мойсик
 2137: (Ростов-на-Дону), Марию Подрядчикову (Волгоград), Павла Солюкова
 2138: (Краснодар), Дмитрия Тарарыкова и Артема Шинкевича (оба - Москва).
 2139: 
 2140: Вопрос 1:
 2141: По легенде, художник ЗЕвксис не смог найти достаточно красивую девушку,
 2142: для того чтобы изобразить Елену Прекрасную. Мэтью ГАмперт сравнивает
 2143: ЗЕвксиса с заглавным героем произведения XIX века. Назовите этого героя.
 2144: 
 2145: Ответ:
 2146: [Виктор] Франкенштейн.
 2147: 
 2148: Комментарий:
 2149: Легенда гласит, что ЗЕвксис отобрал для позирования пятерых девушек и
 2150: позаимствовал лучшие черты каждой из них для портрета Елены. В романе
 2151: Мэри Шелли Франкенштейн создал своего монстра, используя различные
 2152: фрагменты тел.
 2153: 
 2154: Источник:
 2155:    1. Matthew Gumpert. Grafting Helen: The Abduction of the Classical
 2156: Past.
 2157: https://books.google.ru/books?id=5TM1hIAfX-EC&pg=PA258#v=onepage&q&f=false
 2158:    2. http://ru.wikipedia.org/wiki/Виктор_Франкенштейн
 2159: 
 2160: Автор:
 2161: Олег Михеев (Краснодар)
 2162: 
 2163: Вопрос 2:
 2164: Желающие поучаствовать в проводимом в Сахаре марафоне должны оплатить
 2165: вступительный взнос. В русское специализированное название того, для
 2166: чего, помимо прочего, предназначен этот взнос, входит число. Напишите
 2167: это число.
 2168: 
 2169: Ответ:
 2170: 200.
 2171: 
 2172: Комментарий:
 2173: Марафон является достаточно опасным, так что этот взнос предназначен на
 2174: случай смерти участника и репатриации его тела на родину. В вооруженных
 2175: силах тела погибших принято называть "грузом 200".
 2176: 
 2177: Источник:
 2178:    1. https://www.si.com/vault/2002/04/29/8103066/hot--cold-at-the-marathon-des-sables-billed-as-the-worlds-toughest-footrace-even-the-stoutest-of-competitors-couldnt-avoid-getting-saharan-sand-kicked-and-blown-in-their-faces
 2179:    2. http://ru.wikipedia.org/wiki/Груз_200
 2180: 
 2181: Автор:
 2182: Олег Михеев (Краснодар)
 2183: 
 2184: Вопрос 3:
 2185: Рассказывают, что каждый день с февраля 1858 года на протяжении
 2186: четырнадцати лет Бобби можно было встретить на кладбище ГрейфрАйерс в
 2187: Эдинбурге. В статье журнала "National Geographic" [нэшнл джиогрЭфик]
 2188: Бобби называют предшественником... Кого?
 2189: 
 2190: Ответ:
 2191: Хатико.
 2192: 
 2193: Комментарий:
 2194: Скайтерьер Бобби стал знаменит тем, что охранял могилу хозяина, а затем
 2195: был похоронен недалеко от ворот кладбища. Вскоре в Эдинбурге был открыт
 2196: памятник Бобби. Более известна похожая история про японского пса Хатико.
 2197: 
 2198: Источник:
 2199: http://www.nat-geo.ru/fact/38617-predannyy-pes-bobbi/
 2200: 
 2201: Автор:
 2202: Александр Усков (Краснодар)
 2203: 
 2204: Вопрос 4:
 2205: По одной из версий, альбом известной группы был назван в честь команды
 2206: для устройств Macintosh [макинтОш]. Сейчас схожая по структуре команда
 2207: используется в ИКСЕ. На вышеупомянутом альбоме ИКС тоже есть. Назовите
 2208: ИКС одним словом.
 2209: 
 2210: Ответ:
 2211: Android.
 2212: 
 2213: Зачет:
 2214: Андроид.
 2215: 
 2216: Комментарий:
 2217: Фраза "OK Computer", ставшая названием альбомы группы "Radiohead"
 2218: [рэйдиохЭд], была командой для голосового распознавания в Macintosh. В
 2219: операционной системе Android для той же цели используется команда "OK
 2220: Google". Композиция "Paranoid Android" [паранОид андрОид] входит в
 2221: вышеупомянутый альбом.
 2222: 
 2223: Источник:
 2224: http://ru.wikipedia.org/wiki/OK_Computer
 2225: 
 2226: Автор:
 2227: Александр Усков (Краснодар)
 2228: 
 2229: Вопрос 5:
 2230: По официальной версии, в Краснодаре весной этого года ОНО не состоялось
 2231: по соображениям безопасности. Кроме того, представитель епархии упомянул
 2232: негативные ассоциации с древним кельтским обычаем. Назовите ЕГО.
 2233: 
 2234: Ответ:
 2235: Сожжение [чучела] Масленицы.
 2236: 
 2237: Зачет:
 2238: Сжигание [чучела] Масленицы.
 2239: 
 2240: Комментарий:
 2241: Как один из ближайших аналогов русского обычая, представители епархии
 2242: упомянули обряд жертвоприношения друидов, в процессе которого жертва
 2243: заключается в специальную клетку, называемую плетеным человеком, и
 2244: подвергается сожжению.
 2245: 
 2246: Источник:
 2247:    1. https://www.yuga.ru/news/392580/
 2248:    2. http://ru.wikipedia.org/wiki/Плетёный_человек
 2249: 
 2250: Автор:
 2251: Александр Усков (Краснодар)
 2252: 
 2253: Вопрос 6:
 2254: В этом вопросе словом "ИКС" мы заменили другое слово.
 2255:    Одна из статей американского закона устанавливает, в каких случаях
 2256: кража считается совершённой в крупном размере. Например, если сумма
 2257: украденного превышает 950 долларов или если украден ИКС. С этой статьей
 2258: связывают происхождение фразы из трех слов, зачастую сокращаемой до
 2259: аббревиатуры. Назовите эту аббревиатуру.
 2260: 
 2261: Ответ:
 2262: GTA.
 2263: 
 2264: Зачет:
 2265: ГТА.
 2266: 
 2267: Комментарий:
 2268: Выражение, которым названа популярная компьютерная игра, появилось еще в
 2269: середине века. Согласно законам Калифорнии, одним из условий, когда
 2270: кража считается крупной, является угон автомобиля, вне зависимости от
 2271: его цены. Так в уголовный жаргон вошло выражение "Grand Theft Auto"
 2272: [гранд зэфт Ото]. Этим, кстати, объясняется и ненатуральный для
 2273: английского языка порядок слов, ведь более правильным и привычным было
 2274: бы "Auto Grand Theft".
 2275: 
 2276: Источник:
 2277: http://leginfo.legislature.ca.gov/faces/codes_displaySection.xhtml?lawCode=PEN&sectionNum=487
 2278: 
 2279: Автор:
 2280: Олег Михеев (Краснодар)
 2281: 
 2282: Вопрос 7:
 2283: Согласно не вполне серьезному совету, британцам не стоит покупать
 2284: красные автомобили. Дело в том, что заскучавшие полицейские иногда
 2285: соревнуются в так называемой "дорожной" разновидности... Ответьте словом
 2286: английского происхождения: разновидности чего?
 2287: 
 2288: Ответ:
 2289: Снукер.
 2290: 
 2291: Комментарий:
 2292: Снукер - это разновидность бильярда, в который играют 15 красными шарами
 2293: и 6 шарами других цветов, причем забиваться красные и цветные шары
 2294: должны поочередно. Популярная в Англии городская легенда гласит, что
 2295: полицейские иногда играют в снукер машинами, т.е. останавливают машины в
 2296: соответствии с их цветом и правилами снукера, поэтому водители красных
 2297: автомобилей имеют наибольший шанс быть остановленными.
 2298: 
 2299: Источник:
 2300:    1. http://www.exposedpolice.com/uk-traffic-police-playing-motorway-snooker/
 2301:    2. http://ru.wikipedia.org/wiki/Снукер
 2302: 
 2303: Автор:
 2304: Олег Михеев (Краснодар)
 2305: 
 2306: Вопрос 8:
 2307: По одной из версий, причиной тяжелых последствий происшествия в Альпах
 2308: могла стать установленная на НЕМ камера. На НЕМ можно было увидеть семь
 2309: звезд. Назовите ЕГО двумя словами, начинающимися на одну и ту же букву.
 2310: 
 2311: Ответ:
 2312: Шлем Шумахера.
 2313: 
 2314: Комментарий:
 2315: В 2013 году Михаэль Шумахер во время катания на лыжах упал и получил
 2316: серьезную травму головы. Существует версия, что причиной этого могла
 2317: стать камера GoPro [гОу про], установленная на шлеме и разбившая его при
 2318: ударе. Согласно распространенному заблуждению, семь звезд на гоночном
 2319: шлеме Шумахера олицетворяют семь чемпионских титулов "Формулы-1", но на
 2320: самом деле такой дизайн шлема был у него практически с начала карьеры.
 2321: 
 2322: Источник:
 2323:    1. http://www.telegraph.co.uk/sport/motorsport/formulaone/michael-schumacher/10640839/Michael-Schumacher-skiing-crash-did-helmet-camera-cause-head-injuries.html
 2324:    2. http://www.alamy.com/stock-photo-michael-schumacher-a-racing-helmet-of-the-seven-time-formula-1-world-31707308.html
 2325: 
 2326: Автор:
 2327: Олег Михеев (Краснодар)
 2328: 
 2329: Вопрос 9:
 2330: Согласно одному шуточному изображению, ученому удалось создать видимость
 2331: того, что он всё еще работает над НЕЙ. Однако очевидно, что работа уже
 2332: завершена. Назовите ЕЕ словом греческого происхождения.
 2333: 
 2334: Ответ:
 2335: Голограмма.
 2336: 
 2337: Зачет:
 2338: Голография.
 2339: 
 2340: Комментарий:
 2341: Ученый создал голограмму самого себя, которая изображает его за работой.
 2342: 
 2343: Источник:
 2344: http://www.ifunny.com/pictures/greg-was-still-hard-work-hologram-technology/
 2345: 
 2346: Автор:
 2347: Александр Усков (Краснодар)
 2348: 
 2349: Вопрос 10:
 2350: В начале XX века Старый Том помогал китобоям в охоте, взамен получая
 2351: часть добычи. В одной статье это сотрудничество охарактеризовано
 2352: термином, впервые употребленным в современном значении в 1940 году.
 2353: Назовите этот термин словом с удвоенной согласной.
 2354: 
 2355: Ответ:
 2356: Коллаборационизм.
 2357: 
 2358: Комментарий:
 2359: Старый Том был вожаком стаи косаток, сопровождавших усатых китов в
 2360: бухту, где охотились австралийские китобои. Иногда косатки даже хватали
 2361: зубами веревку гарпуна и помогали в буксировке; взамен китобои оставляли
 2362: им языки убитых китов. Коллаборационизм - это осознанное и добровольное
 2363: сотрудничество с врагом. Впервые так стали называть сотрудничавшее с
 2364: немцами правительство ВишИ во Франции.
 2365: 
 2366: Источник:
 2367:    1. http://www.jewishhistoryaustralia.net/Nulla_Nulla_Story/A_5_Killers_in_Eden.htm
 2368:    2. http://en.wikipedia.org/wiki/Collaborationism#Etymology
 2369: 
 2370: Автор:
 2371: Олег Михеев (Краснодар)
 2372: 
 2373: Вопрос 11:
 2374: Журналист и критик Алекс Шепард известен скептическими высказываниями.
 2375: Одно из них привело к тому, что в октябре этого года Шепард опубликовал
 2376: фотографию пластинки Боба Дилана, на которой лежат два металлических
 2377: предмета. Назовите эти предметы.
 2378: 
 2379: Ответ:
 2380: Вилка, нож.
 2381: 
 2382: Зачет:
 2383: В любом порядке.
 2384: 
 2385: Комментарий:
 2386: Боб Дилан регулярно появлялся в списке фаворитов на получение
 2387: Нобелевской премии по литературе. Несмотря на это, Алекс Шепард
 2388: неоднократно заявлял, что не верит в победу Дилана, а в 2015 году даже
 2389: пообещал съесть свой экземпляр пластинки "Blood on the Tracks" [блад он
 2390: зэ трэкс], если это всё же случится.
 2391: 
 2392: Источник:
 2393:    1. https://twitter.com/alex_shephard/status/786530758571724800/
 2394:    2. https://newrepublic.com/article/123058/who-will-win-nobel-prize-literature
 2395: 
 2396: Автор:
 2397: Александр Усков (Краснодар)
 2398: 
 2399: Вопрос 12:
 2400: Проведя раскопки в одном американском городе на улице КонтИ, ученые
 2401: обнаружили, в частности, много румян и бутылок из-под ликера. Это
 2402: позволило предположить, что заведение, упоминаемое в известном
 2403: произведении, действительно существовало. Назовите это произведение.
 2404: 
 2405: Ответ:
 2406: "The House of the Rising Sun" [чтецу: зэ хАус оф зэ рАйзин сан].
 2407: 
 2408: Зачет:
 2409: Дом восходящего солнца.
 2410: 
 2411: Комментарий:
 2412: "Дом восходящего солнца" - народная американская песня. Обычно под ним
 2413: понимают то ли тюрьму, то ли бордель середины XIX века в Новом Орлеане.
 2414: Существовало ли это заведение, до сих пор остается открытым вопросом. По
 2415: словам археолога Шэннон Доуди, проведшей раскопки на предполагаемом
 2416: месте, известном из газет того времени, обстановка выглядела похожей на
 2417: бордель.
 2418: 
 2419: Источник:
 2420:    1. http://en.wikipedia.org/wiki/The_House_of_the_Rising_Sun
 2421:    2. http://www.angelpig.com/house_history.html
 2422: 
 2423: Автор:
 2424: Олег Михеев (Краснодар)
 2425: 
 2426: Тур:
 2427: Финал (Минск). 1 тур
 2428: 
 2429: Дата:
 2430: 25-Mar-2017
 2431: 
 2432: Редактор:
 2433: Ольга Кузьма и Андрей Кузьма (Санкт-Петербург)
 2434: 
 2435: Инфо:
 2436: Редакторы тура благодарят за помощь и ценные замечания Алексея Рабина,
 2437: Максима Веслополова, Евгения и Аллу Муштай (все - Санкт-Петербург) и
 2438: Дмитрия Свинтицкого (Могилев).
 2439: 
 2440: Вопрос 1:
 2441: Из первоначального списка претендентов на звание "музыкальный символ
 2442: чемпионата мира - 2018" сразу были вычеркнуты все духовые инструменты.
 2443: Причиной такого решения стали ОНИ. Назовите ИХ иностранным словом.
 2444: 
 2445: Ответ:
 2446: Вувузелы.
 2447: 
 2448: Комментарий:
 2449: Вувузелы - музыкальный символ чемпионата мира по футболу 2010 года -
 2450: многие до сих пор вспоминают с ужасом. "Вувузела" в переводе с
 2451: зулусского - "делать шум". Музыкальным символом ЧМ-2018 стали ложки.
 2452: 
 2453: Источник:
 2454:    1. http://radiozenit.ru/news_full/uid/621
 2455:    2. http://ru.wikipedia.org/wiki/Вувузела
 2456: 
 2457: Автор:
 2458: Ольга Кузьма, Андрей Кузьма (Санкт-Петербург)
 2459: 
 2460: Вопрос 2:
 2461: Уильяма Фридмана, пытавшегося предупредить ВМФ США о налёте на
 2462: Пёрл-Харбор, называют американским ИМ. ОН родился в 1895 году в
 2463: Бакинской губернии. Назовите ЕГО.
 2464: 
 2465: Ответ:
 2466: [Рихард] Зорге.
 2467: 
 2468: Источник:
 2469: Станислав Зигуненко. 100 великих загадок истории флота. - М.: Вече,
 2470: 2012. - С. 223.
 2471: 
 2472: Автор:
 2473: Ольга Кузьма, Андрей Кузьма (Санкт-Петербург)
 2474: 
 2475: Вопрос 3:
 2476: Адель Алексеева пишет, что на рисунке Пушкина, в отличие от работ многих
 2477: художников, ОН не смешной, а скорее благородный, породистый. Назовите
 2478: ЕГО двумя словами, не используя кавычки.
 2479: 
 2480: Ответ:
 2481: Нос Гоголя.
 2482: 
 2483: Источник:
 2484: Адель Алексеева. Как влюблялись, творили и шалили наши классики. - М.:
 2485: Вече, 2016. - С. 53.
 2486: 
 2487: Автор:
 2488: Ольга Кузьма, Андрей Кузьма (Санкт-Петербург)
 2489: 
 2490: Вопрос 4:
 2491: Известный журналист полагал, что будет похоронен рядом с человеком, чей
 2492: девиз - "Христианство, торговля и цивилизация", но настоятель
 2493: Вестминстерского аббатства воспротивился этому. Назовите фамилию этого
 2494: журналиста.
 2495: 
 2496: Ответ:
 2497: Стэнли.
 2498: 
 2499: Комментарий:
 2500: Слово "полагал" - подсказка. Генри Мортон Стэнли нашел пропавшего в
 2501: Африке Давида Ливингстона и приветствовал его словами "Доктор
 2502: Ливингстон, полагаю?".
 2503: 
 2504: Источник:
 2505:    1. Великие путешествия. Герои, покорившие планету. - СПб.:
 2506: "Пресс-курьер", N 14 за 2016 год. - С. 104.
 2507:    2. http://ru.wikipedia.org/wiki/Ливингстон,_Давид
 2508: 
 2509: Автор:
 2510: Ольга Кузьма, Андрей Кузьма (Санкт-Петербург)
 2511: 
 2512: Вопрос 5:
 2513: Болельщики "Манчестер Юнайтед" дали лидерам своего клуба Полю Погба,
 2514: Златану Ибрагимовичу и Уэйну Руни очень короткое прозвище, намекающее на
 2515: фатальность действий данных футболистов для соперников. Напишите это
 2516: прозвище по-английски.
 2517: 
 2518: Ответ:
 2519: RIP.
 2520: 
 2521: Комментарий:
 2522: Прозвище образовано по первым буквам фамилий игроков. RIP - сокращение
 2523: от "rest in peace" [рест ин пис]. "Покойся с миром" - традиционная
 2524: надпись на надгробьях в англоязычных странах.
 2525: 
 2526: Источник:
 2527: https://www.championat.com/football/news-2543880-trojke-igrokov-mju-runi-ibragimovich-pogba-dali-shutlivoe-prozvische-r-i-p.html
 2528: 
 2529: Автор:
 2530: Ольга Кузьма, Андрей Кузьма (Санкт-Петербург)
 2531: 
 2532: Вопрос 6:
 2533: На Каспии издавна было принято расстилать в прибрежной полосе тряпки и
 2534: старые ковры, прижимая их камнями. Таким образом местные жители ДЕЛАЛИ
 2535: ЭТО для домашнего использования. Россия входят в тройку лидеров среди
 2536: стран, ДЕЛАЮЩИХ ЭТО. Какие слова мы заменили словами "ДЕЛАТЬ ЭТО"?
 2537: 
 2538: Ответ:
 2539: Добывать нефть.
 2540: 
 2541: Источник:
 2542: Станислав Зигуненко. 100 великих загадок истории флота. - М.: Вече,
 2543: 2012. - С. 157.
 2544: 
 2545: Автор:
 2546: Ольга Кузьма, Андрей Кузьма (Санкт-Петербург)
 2547: 
 2548: Вопрос 7:
 2549: В вопросе есть замена.
 2550:    Известный гурман Александр Дюма писал: "Я немного поохотился на
 2551: берегах Каспия, где в таком же изобилии водятся дикие гуси, утки,
 2552: пеликаны, как на Сене - собаки". Восстановите замененное нами слово.
 2553: 
 2554: Ответ:
 2555: Лягушки.
 2556: 
 2557: Источник:
 2558: Великие путешествия. Герои, покорившие планету. - СПб.: Пресс-курьер, N
 2559: 14 за 2016 год. - С. 141.
 2560: 
 2561: Автор:
 2562: Ольга Кузьма, Андрей Кузьма (Санкт-Петербург)
 2563: 
 2564: Вопрос 8:
 2565: Майкл Крайтон пишет, что в период правления бельгийской колониальной
 2566: администрации случаи каннибализма стали более редкими, а к началу 1960-х
 2567: годов в Конго даже появилось несколько ИХ. Одно из НИХ фигурирует в
 2568: заглавии романа 1983 года. Назовите автора этого романа.
 2569: 
 2570: Ответ:
 2571: [Стивен] Кинг.
 2572: 
 2573: Комментарий:
 2574: ОНИ - кладбища. Роман - "Кладбище домашних животных".
 2575: 
 2576: Источник:
 2577:    1. М. Крайтон. Конго. http://flibusta.is/b/135768/read
 2578:    2. http://ru.wikipedia.org/wiki/Кладбище_домашних_животных_(роман)
 2579: 
 2580: Автор:
 2581: Ольга Кузьма, Андрей Кузьма (Санкт-Петербург)
 2582: 
 2583: Вопрос 9:
 2584: Для примата галаго из семейства лориобразных слух не менее важен, чем
 2585: зрение. За внешний вид галаго часто называют именем персонажа,
 2586: появившегося около полувека назад. Назовите этого персонажа.
 2587: 
 2588: Ответ:
 2589: Чебурашка.
 2590: 
 2591: Комментарий:
 2592: У Галаго не только глаза, но и уши велики.
 2593: 
 2594: Источник:
 2595: Дмитрий Бердышев. Самые необычные животные. - М.: ЭНАС-КНИГА, 2016. - С.
 2596: 97, 101.
 2597: 
 2598: Автор:
 2599: Ольга Кузьма, Андрей Кузьма (Санкт-Петербург)
 2600: 
 2601: Вопрос 10:
 2602: Однажды Роберт Вуд растопил несколько килограммов свинца и вылил их в
 2603: небольшое отверстие в земле. После того как свинец застыл, Вуд, потратив
 2604: на раскопки несколько часов, извлек его, получив своеобразный
 2605: скульптурный портрет ЕЕ. Назовите ЕЕ.
 2606: 
 2607: Ответ:
 2608: Молния.
 2609: 
 2610: Комментарий:
 2611: Разветвленная отливка ушла в землю на глубину более трех метров.
 2612: 
 2613: Источник:
 2614: З. Столбовский. Великие тайны и загадки мира. Опасности и угрозы. - М.:
 2615: Мартин, 2005. - С. 156.
 2616: 
 2617: Автор:
 2618: Ольга Кузьма, Андрей Кузьма (Санкт-Петербург)
 2619: 
 2620: Вопрос 11:
 2621: 26 апреля 1945 года Николай Масалов спас ребенка. Через несколько дней в
 2622: его полк приехал человек, поговоривший с Николаем и сделавший несколько
 2623: набросков его внешности. Назовите этого человека.
 2624: 
 2625: Ответ:
 2626: [Евгений] Вучетич.
 2627: 
 2628: Комментарий:
 2629: Впоследствии Вучетич создал знаменитый памятник, установленный в
 2630: Трептов-парке.
 2631: 
 2632: Источник:
 2633: Вячеслав Бондаренко. 100 великих подвигов России. - М.: Вече, 2014. - С.
 2634: 306.
 2635: 
 2636: Автор:
 2637: Ольга Кузьма, Андрей Кузьма (Санкт-Петербург)
 2638: 
 2639: Вопрос 12:
 2640: В 1966 году отряд советских подводных лодок совершил кругосветное
 2641: путешествие, не всплывая на поверхность. За время плавания советские
 2642: моряки за ненадобностью ни разу не совершили некое действие. Шесть лет
 2643: назад в Белоруссии это действие сочли ненужным. Назовите ЕГО.
 2644: 
 2645: Ответ:
 2646: Перевод часов.
 2647: 
 2648: Источник:
 2649:    1. Станислав Зигуненко. 100 великих загадок истории флота. - М.:
 2650: Вече, 2012. - С. 246.
 2651:    2. http://ru.wikipedia.org/wiki/Время_в_Белоруссии
 2652: 
 2653: Автор:
 2654: Ольга Кузьма, Андрей Кузьма (Санкт-Петербург)
 2655: 
 2656: Тур:
 2657: Финал (Минск). 2 тур
 2658: 
 2659: Дата:
 2660: 25-Mar-2017
 2661: 
 2662: Редактор:
 2663: Александр Кудрявцев (Николаев)
 2664: 
 2665: Инфо:
 2666: Редактор благодарит за тестирование вопросов Тараса Вахрива, Дмитрия
 2667: Великова, Анастасию Гончарову, Екатерину Дубровскую, Андрея Кокуленко,
 2668: Юлию Лунёву, Максима Мерзлякова, Дениса Обуха, Дмитрия Овчарука, Алексея
 2669: Рабина, Аркадия Руха и Владимира Шлапака.
 2670: 
 2671: Вопрос 1:
 2672: (pic: 20160954.jpg)
 2673:    На раздаточном материале - рентгеновский ИКС, которым чаще других
 2674: пользовались девушки, имевшие ближневосточные или средиземноморские
 2675: корни. Девушек привлекало то, что эффект держался всю жизнь, а процедура
 2676: была абсолютно безболезненной. Назовите ИКС.
 2677: 
 2678: Ответ:
 2679: Эпилятор.
 2680: 
 2681: Зачет:
 2682: Депилятор.
 2683: 
 2684: Комментарий:
 2685: У девушек с ближневосточной или средиземноморской кровью нередко на лице
 2686: росли усики. С помощью рентгеновского облучения рост волос
 2687: останавливался. Правда, спустя какое-то время начинали проявляться
 2688: нежелательные последствия.
 2689: 
 2690: Источник:
 2691: http://www.cosmeticsandskin.com/cdc/xray.php
 2692: 
 2693: Автор:
 2694: Александр Кудрявцев (Николаев)
 2695: 
 2696: Вопрос 2:
 2697: После того как французский король Филипп I обозвал Вильгельма
 2698: Завоевателя бабой, последний некоторое время ограничивался только
 2699: спиртными напитками, надеясь исправить ситуацию. Какое слово с удвоенной
 2700: согласной мы пропустили в тексте вопроса?
 2701: 
 2702: Ответ:
 2703: Беременной.
 2704: 
 2705: Комментарий:
 2706: Вильгельм сильно растолстел и, после того как Филипп обозвал его
 2707: беременной бабой, решил похудеть, прибегнув к диете, которая состояла
 2708: исключительно из спиртных напитков.
 2709: 
 2710: Источник:
 2711: http://www.history.com/news/history-lists/10-things-you-may-not-know-about-william-the-conqueror
 2712: 
 2713: Автор:
 2714: Александр Кудрявцев (Николаев)
 2715: 
 2716: Вопрос 3:
 2717: Слово "ИКС" является заменой.
 2718:    Мать легендарного налетчика Джесси Джеймса тоже была не промах. После
 2719: смерти сына она организовала прибыльный бизнес, скупая старые ИКСЫ и
 2720: потом продавая их существенно дороже. Английский этимологический словарь
 2721: сообщает, что слово "ИКС" происходит от глагола со значением "свистеть".
 2722: Назовите ИКС.
 2723: 
 2724: Ответ:
 2725: Пистолет.
 2726: 
 2727: Комментарий:
 2728: Мать Джесси Джеймса обманывала покупателей, говоря, что все продаваемые
 2729: ею пистолеты принадлежали ее сыну. Английский этимологический словарь
 2730: указывает на происхождение слова "пистолет" от чешского
 2731: "pi&scaron;t'ala" [пистАла], которое, в свою очередь, происходит от
 2732: глагола "pisteti" [пистЕти] ("свистеть"). Однако если вы подумали, что
 2733: слово происходит от английского глагола "to whistle" [ту висл], - ничего
 2734: страшного.
 2735: 
 2736: Источник:
 2737:    1. http://en.wikipedia.org/wiki/Zerelda_James
 2738:    2. http://www.etymonline.com/index.php?term=pistol&allowed_in_frame=0
 2739: 
 2740: Автор:
 2741: Александр Кудрявцев (Николаев)
 2742: 
 2743: Вопрос 4:
 2744: Когда наивные телезрители стали интересоваться, как вырастить дерево у
 2745: себя дома, компания BBC [би-би-си] ответила, что саженец следует
 2746: посадить в жестянку с НИМ и надеяться на лучшее. Назовите ЕГО двумя
 2747: словами, которые начинаются на соседние буквы.
 2748: 
 2749: Ответ:
 2750: Томатный соус.
 2751: 
 2752: Комментарий:
 2753: В 1957 году в телепрограмме BBC "Панорама" в качестве первоапрельской
 2754: шутки был показан сюжет о необычайно богатом урожае макаронных деревьев
 2755: в Швейцарии.
 2756: 
 2757: Источник:
 2758: http://en.wikipedia.org/wiki/Spaghetti-tree_hoax
 2759: 
 2760: Автор:
 2761: Александр Кудрявцев (Николаев)
 2762: 
 2763: Вопрос 5:
 2764:    <раздатка>
 2765:    низм
 2766:    </раздатка>
 2767:    Существует вероучение, признаЮщее и уважающее все религии, в том
 2768: числе буддизм. Какие буквы мы пропустили в названии этого вероучения?
 2769: 
 2770: Ответ:
 2771: ом.
 2772: 
 2773: Комментарий:
 2774: Ом - широко употребляемая в буддизме мантра. "Omni" [Омни] - латинский
 2775: префикс, означающий "все" или "каждый".
 2776: 
 2777: Источник:
 2778:    1. http://en.wikipedia.org/wiki/Omnism
 2779:    2. http://en.wikipedia.org/wiki/Omni
 2780:    3. http://ru.wikipedia.org/wiki/Ом_(мантра)
 2781: 
 2782: Автор:
 2783: Александр Кудрявцев (Николаев)
 2784: 
 2785: Вопрос 6:
 2786: (pic: 20160955.jpg)
 2787:    Мадрид расположен в предгорьях массива СьЕрра-де-ГвадаррАма. В статье
 2788: об изображенной на раздаточном материале мадридской статуе сообщается и
 2789: ее высота над уровнем моря. Укажите эту высоту.
 2790: 
 2791: Ответ:
 2792: 666 метров.
 2793: 
 2794: Комментарий:
 2795: Это статуя Падшего Ангела в мадридском парке БуЭн-РетИро. Согласно
 2796: христианскому вероучению, падшие ангелы - это ангелы, взбунтовавшиеся
 2797: против Бога и прОклятые им за это. Первым и наиболее известным падшим
 2798: ангелом является Сатана.
 2799: 
 2800: Источник:
 2801: http://en.wikipedia.org/wiki/Fuente_del_%C3%81ngel_Ca%C3%ADdo
 2802: 
 2803: Автор:
 2804: Александр Кудрявцев (Николаев)
 2805: 
 2806: Вопрос 7:
 2807: (pic: 20160956.jpg)
 2808:    Через несколько часов после сооружения изображенная на снимке
 2809: пирамида была сожжена. Назовите двумя словами причину сожжения.
 2810: 
 2811: Ответ:
 2812: Сухой закон.
 2813: 
 2814: Зачет:
 2815: Запрет алкоголя.
 2816: 
 2817: Комментарий:
 2818: Пирамида сооружена из бочек с конфискованным алкоголем, которые через
 2819: несколько минут будут сожжены.
 2820: 
 2821: Источник:
 2822: http://www.kulturologia.ru/blogs/031115/27002/
 2823: 
 2824: Автор:
 2825: Александр Кудрявцев (Николаев)
 2826: 
 2827: Вопрос 8:
 2828: В прошлом веке в США группы добровольцев во избежание трагедий совершали
 2829: поисковые рейды по свалкам и повреждали ИХ, отрывая двери или выламывая
 2830: замки. Назовите ИХ.
 2831: 
 2832: Ответ:
 2833: Холодильники.
 2834: 
 2835: Комментарий:
 2836: В старых холодильниках замок не позволял открыть дверцу изнутри, и дети
 2837: нередко погибали, забравшись внутрь во время игр. Чтобы этого случайно
 2838: не произошло на свалке, добровольцы повреждали выброшенные холодильники.
 2839: 
 2840: Источник:
 2841: http://en.wikipedia.org/wiki/Refrigerator_death
 2842: 
 2843: Автор:
 2844: Александр Кудрявцев (Николаев)
 2845: 
 2846: Вопрос 9:
 2847: В XVII веке английские моряки стали кое-что делать на своем корабле в
 2848: знак скорби по погибшим товарищам, объясняя, что на время траура
 2849: освобождают место для невидимого ИКСА смерти. Так, по одной из версий,
 2850: появилась распространенная современная традиция. Назовите ИКС.
 2851: 
 2852: Ответ:
 2853: Флаг.
 2854: 
 2855: Комментарий:
 2856: Речь идет о традиции во время траура приспускать флаг.
 2857: 
 2858: Источник:
 2859: https://www.washingtonpost.com/news/morning-mix/wp/2015/07/22/a-sign-of-death-not-division-the-bloody-history-behind-lowering-flags-to-half-staff/
 2860: 
 2861: Автор:
 2862: Александр Кудрявцев (Николаев)
 2863: 
 2864: Вопрос 10:
 2865: В 30-е годы в СССР профессия летчика считалась верхом престижа. Поэтому
 2866: в одной телепрограмме утверждалось, что в указанное время мужчина в
 2867: лётной форме во всех смыслах являлся ИМ. Назовите ЕГО двукоренным
 2868: словом.
 2869: 
 2870: Ответ:
 2871: Небожитель.
 2872: 
 2873: Источник:
 2874: Хроники московского быта. Сын Кремля.
 2875: http://www.tvc.ru/channel/brand/id/37/show/episodes/episode_id/45326/
 2876: 
 2877: Автор:
 2878: Александр Кудрявцев (Николаев)
 2879: 
 2880: Вопрос 11:
 2881: Самолет, на котором летает американский президент, называют "Air Force
 2882: One" [эйр форс ван]. Прозвище самолета, на котором летает родившийся в
 2883: 2012 году Американский Фараон, двумя буквами отличается от названия
 2884: президентского самолета. Напишите это прозвище.
 2885: 
 2886: Ответ:
 2887: "Air Horse One" [чтецу: эйр хорс ван].
 2888: 
 2889: Комментарий:
 2890: Американский Фараон - кличка знаменитого скакуна. К местам, где проходят
 2891: скачки, его доставляют специальным самолетом.
 2892: 
 2893: Источник:
 2894:    1. http://www.eonline.com/news/663024/kentucky-derby-champion-american-pharoah-flies-on-a-plane-called-air-horse-one-yes-seriously
 2895:    2. http://en.wikipedia.org/wiki/American_Pharoah
 2896:    3. http://en.wikipedia.org/wiki/Air_Force_One
 2897: 
 2898: Автор:
 2899: Александр Кудрявцев (Николаев)
 2900: 
 2901: Вопрос 12:
 2902: (pic: 20160957.jpg)
 2903:    Поскольку фамилией ИКСА являлось старинное английское слово,
 2904: переводчик Владимир МЕдек использовал старочешское слово с тем же
 2905: значением, имевшее к тому же определенную фонетическую общность.
 2906: Назовите ИКСА.
 2907: 
 2908: Ответ:
 2909: [Альбус] Дамблдор.
 2910: 
 2911: Комментарий:
 2912: На раздаточном материале слово "шмель" написано на старочешском и
 2913: современном чешском языках. Фамилия Альбуса Дамблдора происходит от
 2914: староанглийского слова со значением "шмель"; сравните со словом
 2915: "bumblebee" [бАмблби]. Имя "Альбус" значит "белый", поэтому шмель на
 2916: раздаточном материале - белый.
 2917: 
 2918: Источник:
 2919: http://en.wikipedia.org/wiki/Harry_Potter_in_translation
 2920: 
 2921: Автор:
 2922: Александр Кудрявцев (Николаев)
 2923: 
 2924: Тур:
 2925: Финал (Минск). 3 тур
 2926: 
 2927: Дата:
 2928: 25-Mar-2017
 2929: 
 2930: Редактор:
 2931: Алексей Полевой (Гомель) и Денис Рыбачук (Брест)
 2932: 
 2933: Инфо:
 2934: Редакторы благодарят за помощь в работе над пакетом Максима Мерзлякова
 2935: (Воронеж), Сергея Терентьева и Бориса Моносова (оба - Санкт-Петербург),
 2936: Алексея и Марию Трефиловых (Калуга), Александра Кудрявцева (Николаев),
 2937: Николая Слюняева (Нижний Новгород), Наиля Фарукшина (Навои - Москва),
 2938: Дмитрия Капитанюка (Брест), Максима Новика (Гомель), Александра Огнева
 2939: (Краков), Арсэна Атнагулова (Уфа), Тараса Вахрива (Тернополь), Игоря
 2940: Тюнькина (Москва), Андрея Кокуленко (Омск).
 2941: 
 2942: Вопрос 1:
 2943: В русском переводе одного мультфильма гигант, насмехаясь над
 2944: противником, нараспев произносит обидное слово и повторяет его последний
 2945: слог. Напишите это слово.
 2946: 
 2947: Ответ:
 2948: Блоха.
 2949: 
 2950: Зачет:
 2951: Блоха-ха-ха и т.п.
 2952: 
 2953: Комментарий:
 2954: По сравнению с гигантом противник был очень мал. Переводчика этого
 2955: эпизода, видимо, вдохновила (пауза и ... если есть возможность, то
 2956: поставить музыкальный фрагмент отсюда (только звук, видео не надо) -
 2957: www.youtube.com/watch?v=LXELFcq5qJE с 46 секунды) песня Мефистофеля
 2958: композитора Мусоргского в исполнении Федора Шаляпина.
 2959: 
 2960: Источник:
 2961:    1. Мультфильм "Геркулес" (студия "Дисней", 1997).
 2962:    2. "Блоха" (Песня Мефистофеля в погребке Ауэрбаха), композитор М.П.
 2963: Мусоргский, слова И.В. Гёте, исполняет Ф.И. Шаляпин.
 2964: http://www.youtube.com/watch?v=LXELFcq5qJE
 2965: 
 2966: Автор:
 2967: Денис Рыбачук (Брест)
 2968: 
 2969: Вопрос 2:
 2970: В 1893 году иностранная фирма установила в одном из зданий российского
 2971: города лифт. Лев КЕкушев усовершенствовал его конструкцию и получил
 2972: премию. Что, согласно шутке современников, сделал с лифтом Кекушев?
 2973: 
 2974: Ответ:
 2975: Подковал [его].
 2976: 
 2977: Комментарий:
 2978: Похожая история произошла и с героем Лескова, подковавшим английскую
 2979: блоху. Фирма, правда, была не английская, а немецкая, город - не Тула, а
 2980: Москва, но современники шутили, что премия "за блоху, кою Лёва
 2981: подковал".
 2982: 
 2983: Источник:
 2984: http://moscow-i-ya.livejournal.com/368515.html
 2985: 
 2986: Автор:
 2987: Алексей Полевой (Гомель)
 2988: 
 2989: Вопрос 3:
 2990: В прошлые века при поиске залежей руды разведчики внимательно смотрели
 2991: под ноги во время поездки. Увиденная ОНА, по мнению Тимофея БажЕнова,
 2992: могла привести к открытию месторождения, рядом с которым запылают
 2993: плавильные печи. Назовите ЕЕ.
 2994: 
 2995: Ответ:
 2996: Искра.
 2997: 
 2998: Комментарий:
 2999: Разведчики смотрели под ноги лошади. Подкованное копыто в местах,
 3000: богатых металлическими рудами, могло высечь искру. Если месторождение
 3001: было богатым, то вскоре из искры могло возгореться пламя плавильных
 3002: печей. Стихотворение Пушкина с соответствующей строкой, кстати,
 3003: называется "Во глубине сибирских руд".
 3004: 
 3005: Источник:
 3006: Рейтинг Тимофея Баженова. Человек для опытов. Хозяйка медной горы.
 3007: https://russia.tv/video/show/brand_id/3879/episode_id/170455/
 3008: 
 3009: Автор:
 3010: Денис Рыбачук (Брест)
 3011: 
 3012: Вопрос 4:
 3013: Герои одного романа плохо разбираются в дореволюционных реалиях. Они
 3014: предполагают, что жившие в подвалах рабочие, чтобы выйти на солнечный
 3015: свет, должны были купить ИКС. Какие два слова мы заменили ИКСОМ?
 3016: 
 3017: Ответ:
 3018: Желтый билет.
 3019: 
 3020: Зачет:
 3021: Заменительный билет.
 3022: 
 3023: Комментарий:
 3024: Они думают, что билет желтый, так как по нему можно было выйти на
 3025: солнце. На самом деле желтый билет давал возможность легально заниматься
 3026: проституцией. Редакторы не исключают возможности того, что в зале есть
 3027: игроки, которые знали, что официальным названием "желтого билета" было
 3028: "заменительный билет", и ответили на вопрос, воспользовавшись
 3029: словом-подсказкой "заменили".
 3030: 
 3031: Источник:
 3032:    1. О.Д. Форш. Сумасшедший корабль. http://flibusta.is/b/268668/read
 3033:    2. http://ru.wikipedia.org/wiki/Жёлтый_билет
 3034: 
 3035: Автор:
 3036: Денис Рыбачук (Брест)
 3037: 
 3038: Вопрос 5:
 3039: В биографии Ильи Ильфа отмечается, что в первые дни после переезда в
 3040: Москву журналист искал для себя газету, отдавая предпочтение
 3041: широкоформатным. Это объяснялось отсутствием ЕГО. Какое произведение с
 3042: "НЕГО" начинается?
 3043: 
 3044: Ответ:
 3045: Мойдодыр.
 3046: 
 3047: Комментарий:
 3048: В то время Ильф был беден. Возможно, разыскивая газету пошире, чтобы
 3049: удобнее было укрываться, он повторял строчки недавно написанной
 3050: Чуковским сказки:
 3051:    Одеяло убежало,
 3052:    Улетела простыня,
 3053:    И подушка, как лягушка,
 3054:    Ускакала от меня.
 3055: 
 3056: Источник:
 3057:    1. Александра Ильф "Дом, милый дом".
 3058: http://tfile.co/forum/viewtopic.php?t=840279
 3059:    2. http://www.stihi-rus.ru/1/chukovskiy/14.htm
 3060: 
 3061: Автор:
 3062: Денис Рыбачук (Брест)
 3063: 
 3064: Вопрос 6:
 3065: Строительство началось во время Великой Депрессии, дало работу и помогло
 3066: выжить многим людям. Объект оказался очень дорогим - сооружение обошлось
 3067: в 35 миллионов долларов. Назовите этот объект.
 3068: 
 3069: Ответ:
 3070: Мост "Золотые ворота".
 3071: 
 3072: Комментарий:
 3073: На тот момент "Золотые ворота" оказались действительно "золотыми", за
 3074: период строительства начальная смета выросла вдвое. К сожалению, сейчас,
 3075: находясь в депрессии, многие используют мост совсем не по назначению.
 3076: 
 3077: Источник:
 3078:    1. http://gelio.livejournal.com/223154.html
 3079:    2. http://vse-chudesa.ru/chudesa-sveta-noveyshee-vremya/most-zolotie-vorota-v-san-francisko.html
 3080:    3. http://ru.wikipedia.org/wiki/Золотые_Ворота_(мост)
 3081: 
 3082: Автор:
 3083: Алексей Полевой (Гомель)
 3084: 
 3085: Вопрос 7:
 3086: В армии древних греков ИКСЫ одними из первых вступали в битву. Вскоре
 3087: после Берлинской олимпиады копия "ИКСА" оказалась в Германии. Какое
 3088: слово мы заменили ИКСОМ?
 3089: 
 3090: Ответ:
 3091: Дискобол.
 3092: 
 3093: Комментарий:
 3094: Диск использовался и как дальнее метательное оружие. Адольф Гитлер
 3095: считал статую "Дискобола" образцом для подражания и купил в Италии одну
 3096: из двух сохранившихся копий статуи Мирона. Кстати, статуя появляется в
 3097: начале фильма Лени Рифеншталь "Олимпия".
 3098: 
 3099: Источник:
 3100:    1. Документальный фильм "Происхождение Олимпийских игр" (Olympia -
 3101: The Origins of Games) (2016).
 3102: http://tfile.co/forum/viewtopic.php?t=888142
 3103:    2. http://www.bbc.com/russian/society/2015/04/150408_vert_cul_hitlers_idea_of_the_perfect_body
 3104: 
 3105: Автор:
 3106: Денис Рыбачук (Брест)
 3107: 
 3108: Вопрос 8:
 3109: Сотрудник американского журнала вспоминал, как в один прекрасный день
 3110: увидел ИКСА, который схватил АЛЬФУ быстро, как вспышка, и что обратил
 3111: внимание на контраст между ИКСОМ и АЛЬФОЙ. Назовите ИКСА и АЛЬФУ
 3112: словами, начинающимися на одну и ту же букву.
 3113: 
 3114: Ответ:
 3115: Матрос, медсестра.
 3116: 
 3117: Зачет:
 3118: Моряк, медсестра.
 3119: 
 3120: Комментарий:
 3121: (pic: 20160958.jpg)
 3122:    В один прекрасный день, когда американцы узнали об окончании войны,
 3123: они вышли на улицы, и начались гуляния. Фотограф Альфред Эйзенштадт
 3124: вспоминал, что матрос носился по улице быстро и целовал многих женщин,
 3125: но кадр с медсестрой получился особенно эффектным из-за контраста между
 3126: темной формой матроса и белым халатом медсестры. Дело было в Нью-Йорке,
 3127: а Нью-Йорк, как известно, - город контрастов.
 3128: 
 3129: Источник:
 3130: http://www.kp.by/daily/26401/3277924/
 3131: 
 3132: Автор:
 3133: Алексей Полевой (Гомель)
 3134: 
 3135: Вопрос 9:
 3136: Персонажа современного романа выгоняют из ТАКОГО клуба, после того как
 3137: тот использует банку с краской в качестве импровизированной ЕЕ. Назовите
 3138: ТАКУЮ ЕЕ.
 3139: 
 3140: Ответ:
 3141: Пейнтбольная граната.
 3142: 
 3143: Комментарий:
 3144: Персонаж использовал банку с краской как гранату, но в пейнтбольном
 3145: клубе его находчивость не оценили.
 3146: 
 3147: Источник:
 3148: Т. Пратчетт. Джонни и бомба. http://flibusta.is/b/486418/read
 3149: 
 3150: Автор:
 3151: Денис Рыбачук (Брест)
 3152: 
 3153: Вопрос 10:
 3154: КакАпо по кличке СирОкко нередко участвует в международных
 3155: природоохранных конференциях и выставках. По мнению создателей одного
 3156: документального фильма, СирОкко, в отличие от других какАпо, нельзя
 3157: назвать... Каким?
 3158: 
 3159: Ответ:
 3160: Нелетающим.
 3161: 
 3162: Комментарий:
 3163: Новозеландские какАпо - нелетающие птицы, как и киви, но Сирокко
 3164: является послом своего вида и символом охраны природы в Новой Зеландии,
 3165: поэтому часто летает на различные международные конференции, правда, на
 3166: самолетах.
 3167: 
 3168: Источник:
 3169: Документальный цикл "Дикая природа Новой Зеландии" (Wild New Zealand)
 3170: (2016), 3-я серия. http://tfile.co/forum/viewtopic.php?t=885320
 3171: 
 3172: Автор:
 3173: Денис Рыбачук (Брест)
 3174: 
 3175: Вопрос 11:
 3176: Яркие личинки жука мАйки, стремясь попасть в ИКС, собираются на кончиках
 3177: травинок. Назовите ИКС.
 3178: 
 3179: Ответ:
 3180: Улей.
 3181: 
 3182: Комментарий:
 3183: Нелетающие, но яркие личинки образуют подобие цветка и таким образом
 3184: приманивают пчел. Впоследствии они цепляются за пчелу и отпускают только
 3185: тогда, когда та прилетит в улей. Последствия для пчелиной семьи бывают
 3186: трагичными, так как личинки начинают поедать всё, что найдут в улье.
 3187: 
 3188: Источник:
 3189: Документальный цикл "Яд. Достижение эволюции" (2015), 3-я серия.
 3190: http://tfile.co/forum/viewtopic.php?t=878522
 3191: 
 3192: Автор:
 3193: Денис Рыбачук (Брест)
 3194: 
 3195: Вопрос 12:
 3196: В одном романе детей сначала приучают к взаимовыручке, а уже потом к
 3197: самостоятельности. Поэтому до определенного возраста глаза у детей
 3198: сзади, а потом - спереди. Какое слово в вопросе мы заменили?
 3199: 
 3200: Ответ:
 3201: Пуговицы.
 3202: 
 3203: Комментарий:
 3204: Когда все пуговицы на одежде сзади, то одеться ты можешь только с
 3205: чьей-либо помощью, а когда спереди, то уже и самостоятельно.
 3206: 
 3207: Источник:
 3208: Л. Лоури. Дающий. http://flibusta.is/b/236576/read
 3209: 
 3210: Автор:
 3211: Алексей Полевой (Гомель)
 3212: 
 3213: Тур:
 3214: Финал (Минск). 4 тур
 3215: 
 3216: Дата:
 3217: 25-Mar-2017
 3218: 
 3219: Редактор:
 3220: Серафим Шибанов (Москва), при участии Александра Карясова (Самара)
 3221: 
 3222: Инфо:
 3223: Сделать пакет лучше помогали: Анастасия Белова, Глеб Крутинин, Никита
 3224: Поздняков (все - Пущино), Виктория Бочкарёва, Сергей Кухарев, Дарья
 3225: Макушова, Юрий Мотькин, Регина Шарипова (все - Самара), Александр
 3226: Кудрявцев (Николаев), а также команды "Приматы" и "Слон потрогает тебя"
 3227: (обе - Самара).
 3228: 
 3229: Вопрос 1:
 3230: Компания "Alphabet" [Альфабет] перенесла свой сайт на новый домен,
 3231: который позиционируется как альтернатива .com [дот ком]. По мнению
 3232: автора вопроса, схожим образом в своем логотипе подчеркивает
 3233: разнообразие предоставляемых услуг и другая компания. Назовите эту
 3234: другую компанию.
 3235: 
 3236: Ответ:
 3237: "Amazon".
 3238: 
 3239: Комментарий:
 3240: Новый сайт "Alphabet" [Альфабет] находится по адресу abc.xyz [эй-би-си
 3241: дот икс-вай-зет], т.е., можно сказать, от A [эй] до Z [зет]. В логотипе
 3242: "AmaZon" [амазОн] от буквы A [эй] к букве Z [зет] идет стрелочка.
 3243: 
 3244: Источник:
 3245:    1. https://abc.xyz/
 3246:    2. http://en.wikipedia.org/wiki/Alphabet_Inc.
 3247:    3. https://www.amazon.co.uk/
 3248: 
 3249: Автор:
 3250: Илья Иванов (Путилково)
 3251: 
 3252: Вопрос 2:
 3253: "Гамбург" - единственный футбольный клуб, который играл во всех сезонах
 3254: немецкой Бундеслиги. За это клуб получил прозвище "ОН". Назовите фильм
 3255: 1993 года, в котором можно увидеть более десятка ИХ.
 3256: 
 3257: Ответ:
 3258: "Парк Юрского периода".
 3259: 
 3260: Комментарий:
 3261: Клуб прозвали Динозавром (иногда встречается вариант "Динозавр
 3262: Бундеслиги"). В фильме "Парк Юрского периода" много разных динозавров.
 3263: 
 3264: Источник:
 3265:    1. http://en.wikipedia.org/wiki/Hamburger_SV
 3266:    2. http://en.wikipedia.org/wiki/Jurassic_Park_(film)
 3267: 
 3268: Автор:
 3269: Серафим Шибанов (Москва)
 3270: 
 3271: Вопрос 3:
 3272: В песне группы "25/17" [двадцать пять семнадцать] отмечается, что
 3273: реальность и так достаточно сурова, и говорится, что бояться нужно не
 3274: ВТОРЫХ, а ПЕРВЫХ. Кто создал произведение, в заглавии которого
 3275: фигурируют ПЕРВЫЕ и ВТОРЫЕ?
 3276: 
 3277: Ответ:
 3278: [Константин] Симонов.
 3279: 
 3280: Комментарий:
 3281: Строчка из песни звучит так: "Все боятся мертвых, а нужно живых".
 3282: 
 3283: Источник:
 3284:    1. http://rusrap.org.ru/mp3/2517/text/Cherep_i_kosti.htm
 3285:    2. http://ru.wikipedia.org/wiki/Живые_и_мёртвые_(роман)
 3286: 
 3287: Автор:
 3288: Серафим Шибанов (Москва)
 3289: 
 3290: Вопрос 4:
 3291: Надеемся, этот вопрос будет вам приятен.
 3292:    В конце XIX века Лондон часто называли выражением, которое буквально
 3293: переводится как "Большой ОН". Согласно поговорке, ОН тесно связан... С
 3294: чем?
 3295: 
 3296: Ответ:
 3297: С огнем.
 3298: 
 3299: Комментарий:
 3300: Лондон известен своим смогом, поэтому получил прозвище Big Smoke [биг
 3301: смОук], что буквально переводится как "Большой дым". Согласно поговорке,
 3302: дыма без огня не бывает. В начале вопроса есть отсылка к цитате из "Горя
 3303: от ума" "И дым отечества нам сладок и приятен".
 3304: 
 3305: Источник:
 3306:    1. Дж.Э. Гарднер. Возвращение Мориарти.
 3307: http://flibusta.is/b/264025/read
 3308:    2. http://slovarick.ru/374/
 3309: 
 3310: Автор:
 3311: Серафим Шибанов (Москва)
 3312: 
 3313: Вопрос 5:
 3314: В своей книге Крис Тёрни поступил с погубившим древнюю сосну ученым так,
 3315: как должны были поступить с НИМ. В каком городе ОН жил?
 3316: 
 3317: Ответ:
 3318: Эфес.
 3319: 
 3320: Комментарий:
 3321: Крис Тёрни решил, что не стоит упоминать имя ученого, уничтожившего
 3322: столь ценный объект. Тот, кого древние греки запретили вспоминать, жил в
 3323: Эфесе и сжег местный храм Артемиды.
 3324: 
 3325: Источник:
 3326:    1. К. Тёрни. Кости, скалы и звезды. Наука о том, когда что произошло.
 3327: http://flibusta.is/b/320384/read
 3328:    2. http://en.wikipedia.org/wiki/Herostratus
 3329: 
 3330: Автор:
 3331: Игорь Тюнькин (Москва)
 3332: 
 3333: Вопрос 6:
 3334: Футболист МАриан ЧИшовски окончил карьеру в 2014 году. Незадолго до
 3335: завершения карьеры спортсмен принял участие в ЭТОМ с подачи хоккеиста
 3336: ВАцлава ПлЕтки. Назовите ЭТО тремя английскими словами.
 3337: 
 3338: Ответ:
 3339: Ice Bucket Challenge [чтецу: айс бАкет чЕллендж].
 3340: 
 3341: Зачет:
 3342: Айс бакет челлендж.
 3343: 
 3344: Комментарий:
 3345: Чишовски принял участие в флешмобе под названием "Ice Bucket Challenge"
 3346: [айс бакет чЕллендж], который должен был привлечь внимание к боковому
 3347: амиотрофическому склерозу. К сожалению, через пару месяцев после этого
 3348: Мариан узнал, что сам болен этой болезнью.
 3349: 
 3350: Источник:
 3351: http://www.sports.ru/tribuna/blogs/superratings/1182036.html
 3352: 
 3353: Автор:
 3354: Александр Карясов (Самара)
 3355: 
 3356: Вопрос 7:
 3357: [Ведущему: выделить голосом слово "распалась".]
 3358:    В 1918 году ЕМУ предложили должность в университете города Черновцы.
 3359: Пока ОН думал, ехать или не ехать, Австро-Венгрия распалась, и ОН
 3360: отказался. Назовите ЕГО.
 3361: 
 3362: Ответ:
 3363: [Эрвин] Шрёдингер.
 3364: 
 3365: Комментарий:
 3366: А если бы Австро-Венгрия не распалась, Шрёдингер принял бы это
 3367: предложение. В знаменитом мысленном эксперименте кот жив, если атом
 3368: радиоактивного вещества не распался, и мертв, если распался.
 3369: 
 3370: Источник:
 3371: http://ru.wikipedia.org/wiki/Шрёдингер,_Эрвин
 3372: 
 3373: Автор:
 3374: Александр Карясов (Самара)
 3375: 
 3376: Вопрос 8:
 3377:    <раздатка>
 3378:    В памятник Захер-Мазоху во Львове вмонтирована ОНА, через которую
 3379: можно разглядывать сменяющиеся эротические картинки. ONA изображена на
 3380: памятнике в европейской столице вместе... С кем?
 3381:    </раздатка>
 3382:    Вопрос перед вами. Время!
 3383: 
 3384: Ответ:
 3385: С Ромулом и Ремом.
 3386: 
 3387: Комментарий:
 3388: В первом случае имеется в виду лупа как прибор для улучшения зрения, а
 3389: во втором - Lupa capitolina - "Капитолийская волчица".
 3390: 
 3391: Источник:
 3392:    1. http://www.travel.ru/news/2008/03/24/121343.html
 3393:    2. http://it.wikipedia.org/wiki/Lupa_capitolina
 3394: 
 3395: Автор:
 3396: Александр Карясов (Самара)
 3397: 
 3398: Вопрос 9:
 3399: Советский ученый Владимир СперАнтов рассказывал, как его смелый знакомый
 3400: заявил, что устал от происходящего, и ушел к Сергею. Назовите фамилию
 3401: Сергея.
 3402: 
 3403: Ответ:
 3404: Прокофьев.
 3405: 
 3406: Комментарий:
 3407: Сергей Генкин, как и многие советские люди, присутствовал на похоронах
 3408: Сталина. Однако ему надоела толпа людей, и поэтому он захотел пойти на
 3409: похороны Прокофьева, который также умер 5 марта 1953 года.
 3410: 
 3411: Источник:
 3412: http://050353.ru/2016/03/04/sperantov/
 3413: 
 3414: Автор:
 3415: Александр Карясов, в редакции Юрия Мотькина (оба - Самара)
 3416: 
 3417: Вопрос 10:
 3418: ЕЕ дочерей зовут ОктАвия и СеверИна. В одном интервью ОНА призналась,
 3419: что в детстве была на домашнем обучении. Кто ОНА?
 3420: 
 3421: Ответ:
 3422: [Валерия Гай] Германика.
 3423: 
 3424: Комментарий:
 3425: Создательница сериала "Школа" сама в школу толком и не ходила.
 3426: 
 3427: Источник:
 3428:    1. http://ru.wikipedia.org/wiki/Германика,_Валерия_Гай_Александровна
 3429:    2. http://www.razgovorchiki.ru/arkhiv/germanika.htm
 3430: 
 3431: Автор:
 3432: Серафим Шибанов (Москва)
 3433: 
 3434: Вопрос 11:
 3435: В статье о НЕМ упоминается старовенгерский алфавит, придуманный
 3436: пастухами. Напишите слово, которое мы обозначили как ОН, при помощи
 3437: НЕГО.
 3438: 
 3439: Ответ:
 3440:    Бустро
 3441:    нодеф
 3442: 
 3443: Зачет:
 3444: Часть слова "бустрофедон" должна быть написана в одном направлении,
 3445: часть - в другом.
 3446: 
 3447: Комментарий:
 3448: Бустрофедон - способ письма, при котором чередуются строки, идущие слева
 3449: направо и справа налево.
 3450: 
 3451: Источник:
 3452: http://en.wikipedia.org/wiki/Boustrophedon
 3453: 
 3454: Автор:
 3455: Александр Карясов (Самара)
 3456: 
 3457: Вопрос 12:
 3458: Сериал "Во все тяжкие" повествует об Уолтере Уайте, которому
 3459: диагностируют неоперабельный рак. Вскоре после окончания сериала в
 3460: издании "Albuquerque Journal" [альбукЕрке джОрнал] появился ОН. Назовите
 3461: ЕГО точно.
 3462: 
 3463: Ответ:
 3464: Некролог [Уолтера] Уайта.
 3465: 
 3466: Комментарий:
 3467: Фанаты не поленились, и не важно, что Уолтер Уайт был всего лишь
 3468: персонаж.
 3469: 
 3470: Источник:
 3471: http://variety.com/2013/tv/news/breaking-bad-walter-white-obit-1200694265/
 3472: 
 3473: Автор:
 3474: Серафим Шибанов (Москва)
 3475: 
 3476: Тур:
 3477: Финал (Минск). 5 тур
 3478: 
 3479: Дата:
 3480: 25-Mar-2017
 3481: 
 3482: Редактор:
 3483: Мишель Матвеев (Санкт-Петербург)
 3484: 
 3485: Инфо:
 3486: Редактор благодарит тестеров: Антон Тахтаров, Яна Азриэль, Дмитрий
 3487: Овчарук, Наталья Орлова, Сергей Терентьев, Александр Коробейников,
 3488: Владимир Бройда, Александр Камаев, Юлия Фукельман, Сергей Лобачёв,
 3489: Владимир Городецкий, Ирина Зубкова, Антон Волосатов, Валерий Юдачёв,
 3490: Илья Чадаев, Андрей Данченко, Алексей Акименко, Глеб Олейник, Андрей
 3491: Кокуленко, Ваган Калайджян, Садиг Гамидов, Артем Рожков.
 3492: 
 3493: Вопрос 1:
 3494: Один человек был изуродован своими врагами, но в Петербурге нашелся
 3495: искусный специалист, который ему помог. Исследователь Крашенинников
 3496: усматривает в этом основу сюжета произведения. Какого?
 3497: 
 3498: Ответ:
 3499: "Нос".
 3500: 
 3501: Комментарий:
 3502: Упомянутому человеку враги отрезали нос, а петербургский мастер
 3503: изготовил для него протез. По мнению Крашенинникова, искусственный нос
 3504: мог навеять Гоголю фантазию о самостоятельных прогулках носа коллежского
 3505: асессора Ковалева.
 3506: 
 3507: Источник:
 3508: http://magazines.russ.ru/voplit/2001/5/kpas.html
 3509: 
 3510: Автор:
 3511: Мишель Матвеев (Санкт-Петербург)
 3512: 
 3513: Вопрос 2:
 3514: ОНИ сильно ограничивают свободу действий и поэтому, согласно Википедии,
 3515: стали символом женского целомудрия и мужской власти. Назовите процедуру,
 3516: результатом которой ОНИ становились.
 3517: 
 3518: Ответ:
 3519: Бинтование ног.
 3520: 
 3521: Зачет:
 3522: Забинтовывание ног; перебинтовывание ног; бинтовка ног.
 3523: 
 3524: Комментарий:
 3525: Женщина с ногами, изуродованными бинтованием, была ограничена в
 3526: возможности передвигаться самостоятельно, она была вынуждена сидеть дома
 3527: и не могла пойти куда-либо без сопровождения слуг.
 3528: 
 3529: Источник:
 3530: http://ru.wikipedia.org/wiki/Бинтование_ног
 3531: 
 3532: Автор:
 3533: Мишель Матвеев (Санкт-Петербург)
 3534: 
 3535: Вопрос 3:
 3536: Право посетить непосредственно сам ИКС получает очень небольшое
 3537: количество людей. В частности, один раз это могут сделать нобелевские
 3538: лауреаты. Назовите ИКС двумя словами.
 3539: 
 3540: Ответ:
 3541: Корабль "Ваза".
 3542: 
 3543: Зачет:
 3544: Корабль "Васа".
 3545: 
 3546: Комментарий:
 3547: Музей корабля "Ваза" в Стокгольме ежедневно посещает множество людей, но
 3548: экспозиция расположена вокруг корабля, а сам корабль берегут и
 3549: посетителей на него пускают лишь в исключительных случаях. В частности,
 3550: раз в году после вручения Нобелевской премии лауреаты получают право на
 3551: посещение.
 3552: 
 3553: Источник:
 3554:    1. Рассказ экскурсовода в музее "Ваза".
 3555:    2. http://cr2.livejournal.com/84353.html
 3556: 
 3557: Автор:
 3558: Мишель Матвеев (Санкт-Петербург)
 3559: 
 3560: Вопрос 4:
 3561: К герою Акунина, лежащему в реанимации после автокатастрофы, приходит
 3562: посетитель. Белье героя он называет словом женского рода. Каким?
 3563: 
 3564: Ответ:
 3565: Рубашка.
 3566: 
 3567: Зачет:
 3568: Сорочка.
 3569: 
 3570: Комментарий:
 3571: Идиома "родиться в рубашке" означает "быть везучим, счастливо избегать
 3572: беды". По мнению посетителя, белье героя, выжившего в автокатастрофе, -
 3573: это и есть та рубашка, в которой он родился.
 3574: 
 3575: Источник:
 3576: Б. Акунин. Фантастика. http://flibusta.is/b/182057/read
 3577: 
 3578: Автор:
 3579: Мишель Матвеев (Санкт-Петербург)
 3580: 
 3581: Вопрос 5:
 3582: На знаменитом мероприятии охрана может проверить любого посетителя,
 3583: выходящего из палатки. Так предотвращаются тысячи ИХ. Назовите ИХ тремя
 3584: словами.
 3585: 
 3586: Ответ:
 3587: Кражи пивных кружек.
 3588: 
 3589: Зачет:
 3590: Похищения пивных кружек; кражи/похищения пивных бокалов.
 3591: 
 3592: Комментарий:
 3593: Мероприятие - Октоберфест. Кружки крадут не только ради кружек, но еще и
 3594: просто потому, что это стало популярным развлечением. Приходится
 3595: принимать меры.
 3596: 
 3597: Источник:
 3598: http://ru.wikipedia.org/wiki/Октоберфест
 3599: 
 3600: Автор:
 3601: Мишель Матвеев (Санкт-Петербург)
 3602: 
 3603: Вопрос 6:
 3604: Микробиолог Блэкмор выяснил, что некоторые бактерии могут двигаться
 3605: необычным образом благодаря содержащимся в их организме кристаллическим
 3606: цепочкам железа. Аналогом чего являются эти цепочки?
 3607: 
 3608: Ответ:
 3609: Компаса.
 3610: 
 3611: Комментарий:
 3612: Эти бактерии могут двигаться строго на север или на юг, поскольку
 3613: упомянутые цепочки вытягиваются вдоль линий магнитного поля Земли.
 3614: 
 3615: Источник:
 3616:    1. С.И. Венецкий. В мире металлов. http://flibusta.is/b/224963/read
 3617:    2. http://ru.wikipedia.org/wiki/Магнетосома
 3618: 
 3619: Автор:
 3620: Мишель Матвеев (Санкт-Петербург)
 3621: 
 3622: Вопрос 7:
 3623: Сергей Бубка пошутил, что, когда в одном государстве происходит некое
 3624: событие, стоит такой рев, что его слышно в соседних государствах.
 3625: Назовите это событие.
 3626: 
 3627: Ответ:
 3628: Гран-при Монако.
 3629: 
 3630: Зачет:
 3631: Гонки "Формула-1" и т.п. по словам "Формула-1".
 3632: 
 3633: Комментарий:
 3634: Имеется в виду рев моторов. Монако - совсем маленький анклав внутри
 3635: Франции, а расстояние до итальянской границы составляет всего десяток
 3636: километров. Сергей Бубка, как и ряд других спортсменов, по окончании
 3637: карьеры поселился в Монако.
 3638: 
 3639: Источник:
 3640: https://esquire.ru/wil/bubka
 3641: 
 3642: Автор:
 3643: Мишель Матвеев (Санкт-Петербург)
 3644: 
 3645: Вопрос 8:
 3646: Дуплет.
 3647:    1. Персонаж одного романа называл ЕЕ Венерой. Назовите ЕЕ двумя
 3648: словами.
 3649:    2. Персонажу классического произведения в критический момент
 3650: показалось, что ОНА - это аристократка. Назовите ЕЕ двумя словами.
 3651: 
 3652: Ответ:
 3653:    1. Дама червей.
 3654:    2. Дама пик.
 3655: 
 3656: Комментарий:
 3657:    1. Венера - богиня любви, сердце - символ любви и символ червовой
 3658: масти.
 3659:    2. В "Пиковой даме" Германну в момент проигрыша померещилось, что
 3660: дама пик была старухой-графиней.
 3661: 
 3662: Источник:
 3663:    1. Б. Акунин. Азазель. http://flibusta.is/b/228606/read
 3664:    2. А.С. Пушкин. Пиковая дама.
 3665: http://www.rvb.ru/pushkin/01text/06prose/01prose/0866.htm
 3666: 
 3667: Автор:
 3668: Мишель Матвеев (Санкт-Петербург)
 3669: 
 3670: Вопрос 9:
 3671: Гравитон - это гипотетическая частица, переносящая гравитационное
 3672: взаимодействие. Часто задаются вопросом, почему ОН не оказывается
 3673: непреодолимым для гравитонов. Назовите ЕГО.
 3674: 
 3675: Ответ:
 3676: Горизонт событий [черной дыры].
 3677: 
 3678: Комментарий:
 3679: С одной стороны, черная дыра гравитационно взаимодействует с окружающим
 3680: миром. С другой стороны, согласно общей теории относительности, ничто, в
 3681: числе гравитон, не может выбраться из-за ее горизонта событий. Есть
 3682: несколько возможных способов разрешения этого противоречия - например,
 3683: гравитация может распространяться виртуальными гравитонами, рождающимися
 3684: вблизи горизонта событий. Окончательный ответ, видимо, зависит от того,
 3685: какой будет теория, в рамках которой будет описан гравитон.
 3686: 
 3687: Источник:
 3688:    1. https://www.quora.com/Can-gravitons-escape-the-event-horizon-of-a-black-hole-If-not-how-can-we-detect-the-black-holes-gravity
 3689:    2. http://physics.stackexchange.com/questions/107185/are-gravitons-bound-by-the-event-horizon
 3690:    3. http://curious.astro.cornell.edu/physics/89-the-universe/black-holes-and-quasars/theoretical-questions/451-how-do-gravitons-escape-black-holes-to-tell-the-universe-about-their-gravity-advanced
 3691:    4. https://www.google.ru/?gws_rd=ssl#q=graviton+event+horizon
 3692: 
 3693: Автор:
 3694: Мишель Матвеев (Санкт-Петербург)
 3695: 
 3696: Вопрос 10:
 3697: Согласно несерьезному мнению, это произведение следует признать
 3698: экстремистским за описание убийства главы государства при помощи дрона.
 3699: Назовите это произведение.
 3700: 
 3701: Ответ:
 3702: "Сказка о золотом петушке".
 3703: 
 3704: Комментарий:
 3705: Золотой петушок, убивший царя Додона, - вполне себе дрон. А домик Элли,
 3706: если что, не был управляемым, да и Гингема - не глава государства.
 3707: 
 3708: Источник:
 3709: http://www.anekdot.ru/id/822764/
 3710: 
 3711: Автор:
 3712: Мишель Матвеев (Санкт-Петербург)
 3713: 
 3714: Вопрос 11:
 3715: Одна из версий объясняет происхождение известного спортивного термина
 3716: так: он образован от французского слова "l'&oelig;uf" [лёф], означающего
 3717: яйцо, а яйцо похоже на... Что?
 3718: 
 3719: Ответ:
 3720: Ноль.
 3721: 
 3722: Комментарий:
 3723: В теннисе ноль очков почему-то называют словом "love" [лав]. Причины и
 3724: пытается объяснить приведенная версия.
 3725: 
 3726: Источник:
 3727: http://en.wikipedia.org/wiki/Tennis_scoring_system
 3728: 
 3729: Автор:
 3730: Мишель Матвеев (Санкт-Петербург)
 3731: 
 3732: Вопрос 12:
 3733: В рассказе об известном произведении говорится, что в Карлсбаде некто
 3734: пошутил: "Видите, как действуют на организм карлсбадские воды!".
 3735: Назовите это произведение.
 3736: 
 3737: Ответ:
 3738: "Прощальная симфония".
 3739: 
 3740: Комментарий:
 3741: По версии шутника, музыканты один за другим уходили со сцены по личным
 3742: мотивам. Кстати, исполнялась не сама "Прощальная симфония" Гайдна, а
 3743: произведение Рихарда Штрауса, использовавшее ту же идею постепенного
 3744: ухода музыкантов.
 3745: 
 3746: Источник:
 3747: http://www.blagaya.ru/put/articles/muzhumor/
 3748: 
 3749: Автор:
 3750: Мишель Матвеев (Санкт-Петербург)
 3751: 

FreeBSD-CVSweb <freebsd-cvsweb@FreeBSD.org>